Sunteți pe pagina 1din 60

Evidence Outline Fall 2012

Introduction to Evidence
I. FRE 201- Judicial Notice of Adjudicative Facts
(a) Scope of rule.
o This rule governs only judicial notice of adjudicative facts.
(b) Kinds of facts.
o A judicially noticed fact must be one not subject to reasonable dispute in that
it is either (1) generally known within the territorial jurisdiction of the trial
court or (2) capable of accurate and ready determination by resort to sources
whose accuracy cannot reasonably be questioned.
(c) When discretionary.
o A court may take judicial notice, whether requested or not.
(d) When mandatory.
o A court shall take judicial notice if requested by a party and supplied with
the necessary information.
(e) Opportunity to be heard.
o A party is entitled upon timely request to an opportunity to be heard as to
the propriety of taking judicial notice and the tenor of the matter noticed.
In the absence of prior notification, the request may be made after judicial
notice has been taken.
(f) Time of taking notice.
o Judicial notice may be taken at any stage of the proceeding.
(g) Instructing jury.
o In a civil action or proceeding, the court shall instruct the jury to accept as
conclusive any fact judicially noticed. In a criminal case, the court shall
instruct the jury that it may, but is not required to, accept as conclusive any
fact judicially noticed.

II. FRE 611. Mode and Order of Interrogation and Presentation


(a) Control by court.
o The court shall exercise reasonable control over the mode and order of
interrogating witnesses and presenting evidence so as to (1) make the
interrogation and presentation effective for the ascertainment of the truth, (2)
avoid needless consumption of time, and (3) protect witnesses from
harassment or undue embarrassment.
Restates in broad terms the power and obligation of the judge as
developed under common law principles. Judge has lots of
discretionary power. Can decide form, the pace, presentation of
evidence, the many other questions arising during the course of a trial
which can be solved only by the judge's common sense and fairness in
view of the particular circumstances
(b) Scope of cross-examination.
o Cross-examination should be limited to the subject matter of the direct
examination and matters affecting the credibility of the witness. The court
may, in the exercise of discretion, permit inquiry into additional matters as if
on direct examination.
Primary policy consideration: economy of time and energy.
Additional matters = matters bearing upon the credibility of the
witness. Also for those situations in which the result otherwise would
be confusion, complication, or protraction of the case.
(c) Leading questions.
o Leading questions should not be used on the direct examination of a witness
except as may be necessary to develop the witness' testimony. Ordinarily
leading questions should be permitted on cross-examination. When a party
calls a hostile witness, an adverse party, or a witness identified with an
adverse party, interrogation may be by leading questions.

III. FRE 607. Who May Impeach


The credibility of a witness may be attacked by any party, including the party calling
the witness.
o Abandons the old common law rule, denial of the right leaves the party at the
mercy of the witness and the adversary.

Relevancy
I. Rule 104. Preliminary Questions
(a) Questions of admissibility generally.
o Preliminary questions concerning the qualification of a person to be a witness,
the existence of a privilege, or the admissibility of evidence shall be
determined by the court, subject to the provisions of subdivision (b). In
making its determination it is not bound by the rules of evidence except those
with respect to privileges.
Judge acts as the trier of fact in this case, and evaluates the legally set
standards for admissibility.
(b) Relevancy conditioned on fact.
o When the relevancy of evidence depends upon the fulfillment of a condition
of fact, the court shall admit it upon, or subject to, the introduction of
evidence sufficient to support a finding of the fulfillment of the condition.
Aka Conditional relevancy. The judge makes a preliminary
determination whether the foundation evidence is sufficient to support
a finding of fulfillment of the condition. If so, the item is admitted.
The jury then decides whether the fulfillment of the condition is
established.
(c) Hearing of jury.
o Hearings on the admissibility of confessions shall in all cases be conducted
out of the hearing of the jury. Hearings on other preliminary matters shall be
so conducted when the interests of justice require, or when an accused is a
witness and so requests.
(d) Testimony by accused.
o The accused does not, by testifying upon a preliminary matter, become subject
to cross-examination as to other issues in the case.
The limitation upon cross-examination is designed to encourage
participation by the accused in the determination of preliminary
matters.
(e) Weight and credibility.
o This rule does not limit the right of a party to introduce before the jury
evidence relevant to weight or credibility.

II. Rule 105. Limited Admissibility


When evidence which is admissible as to one party or for one purpose but not admissible
as to another party or for another purpose is admitted, the court, upon request, shall restrict the
evidence to its proper scope and instruct the jury accordingly.
Closely related to FRE 403 (Exclusion of relevant evidence based on prejudice,
waste, etc.). This rule must be taken into consideration when making a 403
determination.
See what is in issue in the case, and whether or not the evidence being brought in
relates to the issue in the case. Ex: In a case where an accident cause damage to a
right hand, testimony that the person was flexing her left hand back and forth is
irrelevant.

III. FRE 401. Definition of "Relevant Evidence"


"Relevant evidence" means evidence having any tendency to make the existence of any
fact that is of consequence to the determination of the action more probable or less probable than
it would be without the evidence.
Problems of relevancy call for an answer to the question whether an item of evidence,
when tested by the processes of legal reasoning, possesses sufficient probative value
to justify receiving it in evidence.
Relevancy is not an inherent characteristic of any item of evidence but exists only as
a relation between an item of evidence and a matter properly provable in the case
The standard of probability under the rule is "more * * * probable than it would be
without the evidence

3 Questions for Relevance


(1) What is evidence being used to prove?
(2) Is that proposition provable in the case?
(3) Does it help in proving or disproving that proposition?

IV. FRE 401 in Depth


A. 401 contains two elements combined into one rule.
1. Materiality to be admissible, all evidence must be material, i.e. offered to prove a
properly provable issue in the case. In other words, what is the evidence trying to
show and is this an issue in the case?
2. Probative value- Evidence must logically tend to prove the proposition for which it
was offered. Evidence need not be determinative; it need only make the fact to which
it is directed more likely than it would be without the evidence. If the evidence would
support an inference of the ultimate fact to be proven, it is probably sufficiently
probative to be relevant.
B. Degree of Probativeness necessary is a low bar. Only needs to tend to prove.
1. Knapp v. State- Knapp (D) killed someone and claimed self-defense. D testified
that he had head the victim had previously killed an old man. Prosecution had a
doctor testify that the old man died of senility and alcoholism and defendant did not
want that evidence in.
o a. Competency of a collateral fact is not to be determined by the
conclusiveness of the inferences it may afford. It is enough if these may tend
in a slight degree to elucidate the inquiry, or to assist to a determination
probably founded in truth. Evidence of the death by natural causes was
admitted.
C. Evidence needs to address an issue in the case.
2. Sherod v. Barry- Robbery suspect shot by the police when he made a quick
movement with his hand into his coat. Later found to be unarmed, and this fact was
presented at trial. D objected.
o a. Any information beyond that which Officer Barry had and reasonably
believed at the time he fired his revolver is improper, irrelevant and
prejudicial to the determination whether the officer acted reasonably under the
circumstances.
D. Knowledge of Weakness of Case is relevant- McQueeny v. Willmington
1. P injury on a ship, convinced a friend to commit perjury in a depo to support his
case. P said they werent going to use Depo, D said they wanted to introduce it.
2. Depo would show that P had knowledge of the weakness of his case therefore
willing to commit perjury.
o a. Evidence of subornation of perjury is substantive evidence, not mere
impeachment material.
o Is the evidence of the list and deposition admissible at trial? Is it relevant? The
judge doesnt think it is relevant because the plaintiff said that they will not
call Cerda and offer the deposition, so they do not have to go down the path of
looking at this evidence. The broader argument is that McQueeney did not
perjured himself and the issue is about him falling, so this is not relevant to
the case.

V. Rule 402. Relevant Evidence Generally Admissible; Irrelevant Evidence Inadmissible


All relevant evidence is admissible, except as otherwise provided by the Constitution of
the United States, by Act of Congress, by these rules, or by other rules prescribed by the
Supreme Court pursuant to statutory authority. Evidence which is not relevant is not admissible.
Other Rules that can govern the admissibility of evidence: the Rules of Civil and
Criminal Procedure, by Bankruptcy Rules, by Act of Congress, or by constitutional
considerations.
VI. FRE 402 in Depth
A. Relevancy is the basic underlying concept in evidence.
1. Nothing is to be received which is not logically probative of some matter requiring
to be proved.
2. Everything which is thus probative should come in, unless a clear ground of policy
or law excludes it.
B. Circumstantial Evidence is relevant and is left to the trier to fact to decide the strength
of inferences it affords. Stevenson v. Stewart
1. Forgery on a debt of a bill (promissory note). Action of debt on a bill dated July 6th,
1844. The defendant pleaded forgery. The defendant produced witnesses that stated
that he was not in the country. A plaintiff offered a witness who stated he loaned
defendant $40, and took his note on June 1844.
2. Evidence must correspond w/ the allegations and be confined to the point at issue.
But dont need a direct connection, great latitude is allowed for indirect, or
circumstantial evidence . Enough that it has a tendency to prove.
3. Convincing power of inference is up to the jury to decide. It is sufficient that
evidence has some affinity with the principal inquiry though weak or remote.

VII. Rule 403. Exclusion of Relevant Evidence on Grounds of Prejudice, Confusion, or


Waste of Time
Although relevant, evidence may be excluded if its probative value is substantially
outweighed by the danger of unfair prejudice, confusion of the issues, or misleading the jury, or
by considerations of undue delay, waste of time, or needless presentation of cumulative evidence.
Some risks of relevant evidence range all the way from inducing decision on a purely
emotional basis, at one extreme, to nothing more harmful than merely wasting time,
at the other extreme.
This rule calls for balancing the probative value of and need for the evidence against
the harm likely to result from its admission.
"Unfair prejudice" within its context means an undue tendency to suggest decision on
an improper basis, commonly, though not necessarily, an emotional one.
In reaching a decision whether to exclude on grounds of unfair prejudice,
consideration should be given to the probable effectiveness or lack of effectiveness of
a limiting instruction.

VIII. Rule 403 in Depth


A. Judges generally have broad discretion with 403 rulings. Appellate court will reverse
only if judge abuses discretion. Factors to determine:
1. the importance of the fact of consequence
2. the strength in the chain of inferences
3. the alternative means of proof
4. whether the fact of consequence is disputed
o a. disputed: more probative value
o b. not disputed: less probative value
5. the effect of a limiting instruction
B. Exclusion of Relevant Evidence- Old Chief v. US
1. D convicted under statute that made it criminal for felons to carry guns. Before trial
D moved for an order preventing P from mentioning his previous criminal conviction
except to stipulate that he had been convicted of a crime punishable by imprisonment
for 1 year. Judge allowed it and D appeals.
2. Normally, prosecution entitled to prove its case by evidence of its own choice.
(underlying policy: narrative richness, cant stipulate away force of the case). 403,
however, places restrictions on this concept.
3. Unfair prejudice- relevant evidence to lure fact finder into declaring guilt on
ground different from proof specific to the offense charged. (improper basis- usually
emotional)
o a. Typically a state may not present Ds prior troubles, denial of a fair
opportunity to defend against this particular charge.
4. In this case the evidence of past crime was too prejudicial and a stipulation would
have a viable alternative with equal probative value, but less prejudicial effect.
C. Trial Judge does not assess credibility under 403 Ballou v. Henri Studios
1. Judge excluded evidence of toxicology report because Nurse said she didnt smell
alcohol on driver.
2. Trial court made a credibility choice between the test result and nurses testimony.
This was a mistake b/c the probative value to be considered under 403 is the
probative value w/ respect to a material fact if the evidence is believed, not the degree
to which the court finds it believable. It is the jurys role to determine credibility.
3. It is not the judges role to determine credibility of evidence. It is up to the jury to
decide if there is greater weight on the testimony rather than the test.
D. Evidence of perjury has a high probative value.
1. McQueeny v. Willmington (deposition case, supra) - In this case, judge gave
evidence no probative value and lots of prejudice.
o a. Not unfair prejudice. Danger of influence was not outweighed by the
prejudice.
E. All evidence is prejudicial, 403 only excludes unfairly prejudicial evidence. Carter
v Hewitt
1. Carter, an inmate, sued saying that he was beaten by guards. D introduced letter by
P talking about set ups.
2. The ruling is that the difference in inferences in the case should be decided to the
jury. The evidence is relevant and in regards to prejudice, they say that Rule 403
applies to evidence that is unfairly prejudicial and not evidence that is simply
prejudicial.

Subsequent Remedial Measures


I. Rule 407. Subsequent Remedial Measures
When, after an injury or harm allegedly caused by an event, measures are taken that, if
taken previously, would have made the injury or harm less likely to occur, evidence of the
subsequent measures is not admissible to prove negligence, culpable conduct, a defect in a
product, a defect in a product's design, or a need for a warning or instruction. This rule does not
require the exclusion of evidence of subsequent measures when offered for another purpose, such
as proving ownership, control, or feasibility of precautionary measures, if controverted, or
impeachment.
Rationale: Social policy of encouraging people to take, or at least not discouraging
them from taking, steps in furtherance of added safety.
Exclusion is called for only when the evidence of subsequent remedial measures is
offered as proof of negligence or culpable conduct. In effect it rejects the suggested
inference that fault is admitted. Other purposes are, however, allowable, including
ownership or control, existence of duty, and feasibility of precautionary measures, if
controverted, and impeachment.
But if a third party creates the subsequent remedial measure, this may be brought into
evidence because it is relevant.

II. FRE 407 in depth


A. Subsequent Conduct is not an admission/Public Policy to encourage subsequent
remedial measures - Tuer v.McDonald
1. Tuer (P)s husband died and sued the surgeon for malpractice (D). As a result the
hospital changed the protocol with respect to discontinue the use of Heparin for
patient w/ husbands condition.
Another argument is that it is feasible to administer the Heparin to the point of
surgery. The door would be open for plaintiff to offer evidence that it was feasible to
restart the Heparin and continue it until surgery. But the doctor never said it wasnt
feasible, so this evidence would not be admissible. Because they made a decision for
a variety of reasons, they are not controverting feasibility, they are saying they did it
for another reason.
2. Feasibility- construed narrowly, doesnt have to be physically, technologically, or
economically impossible. It is enough to balance the risks of alternatives.
3. Impeachment exception is interpreted narrowly by the courts and is not normally
admissible if offered for simple contradiction of the witness testimony. Not allowed
as a pretext for using the evidence to establish culpability.
o a. If the defendant said it was literally impossible to give Heparin, the change
in protocol might have been admissible for impeachment purposes.

III. Rule 408. Compromise and Offers to Compromise


(a) Prohibited uses. Evidence of the following is not admissible on behalf of any
party, when offered to prove liability for, invalidity of, or amount of a claim that was
disputed as to validity or amount, or to impeach through a prior inconsistent statement
or contradiction:
o (1) furnishing or offering or promising to furnish or accepting or offering or
promising to accept a valuable consideration in compromising or attempting
to compromise the claim ; and
o (2) conduct or statements made in compromise negotiations regarding the
claim, except when offered in a criminal case and the negotiations related to a
claim by a public office or agency in the exercise of regulatory, investigative,
or enforcement authority.
Permitted uses. This rule does not require exclusion if the evidence is offered for
purposes not prohibited by subdivision (a). Examples of permissible purposes include
proving a witness's bias or prejudice ; negating a contention of undue delay; and
proving an effort to obstruct a criminal investigation or prosecution.
o Rationale
(1) The evidence is irrelevant, since the offer may be motivated by a
desire for peace rather than from any concession of weakness of
position.
(2) promotion of the public policy favoring the compromise and
settlement of disputes
o Since the rule excludes only when the purpose is proving the validity or
invalidity of the claim or its amount, an offer for another purpose is not within
the rule.

IV. FRE 408 in Depth


A. Exclusionary rule only applies to real offers to compromise or statements made in the
course of settlement offers. Davidson v. Prince
1. Steer charged P, D wanted to introduce a letter contradicting the claimed distance
between P and the bull.
2. This letter was not an offer to compromise, nor was it written as a part of
settlement negotiations. It was an exchange of information and demand letter. So
lower court was correct to admit it.

V. Guilty Pleas
A. When D enters a guilty plea in a criminal case, it may be introduced in a subsequent
civil trial. However the D may explain the circumstances surrounding the plea. If the guilty plea
is not accepted by the court, it may not be admitted in a subsequent trial.

Hearsay
I. FRE 801. Definitions. Part I.
The following definitions apply under this article:
(a) Statement.
o A "statement" is (1) an oral or written assertion or (2) nonverbal conduct of a
person, if it is intended by the person as an assertion.
The effect of the definition of "statement" is to exclude from the
operation of the hearsay rule all evidence of conduct, verbal or
nonverbal, not intended as an assertion. The key to the definition is
that nothing is an assertion unless intended to be one.
(b) Declarant.
o A "declarant" is a person who makes a statement.
(c) Hearsay.
o "Hearsay" is a statement, other than one made by the declarant while
testifying at the trial or hearing, offered in evidence to prove the truth of the
matter asserted.
If the significance of an offered statement lies solely in the fact that it
was made, no issue is raised as to the truth of anything asserted, and
the statement is not hearsay.
The effect is to exclude from hearsay the entire category of "verbal
acts" and "verbal parts of an act," in which the statement itself affects
the legal rights of the parties or is a circumstance bearing on conduct
affecting their rights.

II. FRE 801 Part I in Depth-


A. Basic Definition
1. 3 basic elements to hearsay
o a. An out of (THIS) court statement
o b. A statement by declarant that is either written, oral, or is conduct meant to
be a statement
o c. Offered for the truth of the matter asserted.
2. Traditional (Wigmore Definition)
o a. Oral statements or
o b. Documents
o c. Assertive Conduct
3. Morgan- includes everything in traditional but also adds nonassertive conduct, or
conduct not intended by the actor as an assertion. For our purposes, Traditional
controls.
B. Rationale- concern about the trustworthiness of hearsay evidence. The reliability of
such evidence is questionable because it was not given under oath, was not given where the fact
finder could observe the declarants demeanor, and was not subject to cross to test perception,
memory, veracity or articulateness of the out of court declarant. See generally: Tribe,
Triangulating Hearsay p. 184-186.

Hearsay risks
Whenever you have a witness to the event, the following risks are involved in the
probative value of the testimony:
o Memory- May be at issue
o Perception- Did they perceive the situation correctly? Was it certain that it was
Johns car?
o Narration- Was the declarant misunderstood in what he or she said?
o Sincerity- Is there fabrication going on here?
But what of witness testifying? Do the same risks exist? Yes- but trial process lessens risk
because (1) it is done under oath, (2) cross-examination can test these risks, and (3)
witnesses demeanor can be looked at by the trier of fact.

Personal Knowledge
Personal knowledge is foundational. If it comes to a situation where the witness is
testifying to facts, you must see if the person has firsthand knowledge of the facts. If
there are facts that establish the witness did not see it or relied on third party reports, then
you know there is problem with the evidence. If the statements are being offered for the
truth, it is a hearsay issue. If they heard a conversation, this will be lack of personal
knowledge as opposed to being a witness to an event.
Eliminate hearsay issue by calling the declarant up as a witness. If you have a witness
testifying that there were in the place at the time and saw Johns car running the red light,
there is not a problem of hearsay. Testifying from personal knowledge is okay. But if the
witness herself refers to her own out of court statement, there is a hearsay problem.

State v. English (1931)


Steven English was convicted of murdering his wife and he appeals because he had
evidence that David Locke admitted to the murder and knew facts about the condition of
the body. Locke was let free and his location was unknown.
The trial court said that since he wasnt there this was hearsay evidence and not
admissible. The evidence in question was evidence was that David Locked admitted to
the murder.
The struggle is that the evidence casts doubt that the person is guilty, balanced with the
rule of law and this is hearsay evidence. The judge had no case but to not admit it. The
precedent was that this evidence was hearsay and they could not admit it under common
law.

III. FRE 801 (c) in depth - Statements that are not necessarily offered for the truth of the
matter asserted and can be introduced as circumstantial evidence (NOT substantive) for whatever
they are offered to prove. aka Non-Hearsay
A. Declarants ability to speak- Estate of Murdock
1. Plane accident victims, children fighting over will. Trial court excluded testimony
by a deputy sheriff at the scene who heard Mr. Murdock whisper Im alive.
2. Court erred because the evidence was offered not for the truth of the statement, but
for the fact a statement was made dead men do not talk.
3. Police officer has personal knowledge. It is not offered for the truth of the matter
asserted, just offering to prove that he survived, which goes to the heart of the issue.
B. Statements offered to show effect on hearer- Duress Subramanian v. Public
Prosecutor
1. Evidence of statements made to a particular person may be offered to show their
state of mind in the sense that they had notice, knowledge, motive, good faith, duress
probably cause, or that they had acquired information that had a bearing on their
subsequent conduct. Not hearsay because not offer for ToM, but to show effect on
hearer.
2. Ds defense was that he was captured by terrorists and under duress.
3. D was not trying to prove that the terrorists would actually harm him if he failed to
abide by their wishes, but only that statements were made that might reasonably have
induced D an apprehension of death is he failed to conform to their wishes.
4. The evidence is admissible. It wasnt important that the terrorists statements were
truthful, but just that it made the defendant think they were true. It was the duress
defense, a state of mind defense. The evidence doesnt go to the truth of the matter
asserted; it was admitted to show that these statements were made and thus made him
think he would be killed. It was a non-hearsay purpose.
C. Proof of Ds Knowledge or Notice- Vinyard v. Vinyard Funeral Home
1. Slip and fall. At the trial, P was allowed to produce testimony from an officer of D
and an employee of D that they had heard complaints from others that the ramp was
slippery when wet. P offered the testimony to show that D had knowledge of the
slickness.
2. The hearsay rule does not apply when the fact that a statement was made is
relevant, regardless of whether the statement is true or false.
3. The complaints made about the slippery pavement is in issue. They were offered to
prove that they knew the state of the parking lot, not that it was in fact slippery. It was
offered for a non-hearsay notice. Notice or knowledge for a fact or condition is a non-
hearsay use of the evidence. Additional evidence must be given to prove slipperiness
D. Statements offered to prove availability of other statements- Johnson v. Misercordia
1. P sued D for negligently hiring incompetent doctor and allowing him to operate. At
trial, introduced records from credentials committees containing medical opinions.
2. Even if the evidence contained medical opinions, they were offered not to prove
the truth of the opinions but to show that the opinions existed and should have been
considered by D.
3. D could have been requested a limiting instruction so that the evidence would not
be considered to establish the doctors incompetence.
4. The truth of the information contained in the out of court records and statements
are introduced to show that there was information available where defendant could
have obtained knowledge of the doctor in question and should have looked at this
evidence. For failing to do so, they committed an act of negligent hiring. The court
affirmed the trial courts ruling of admitting the evidence into the case.
E. Legally Operative Facts (Legally Operative Language)- Words that are legally
significant in and of themselves, regardless of their veracity are admissible. Rise Biological Inc.
v. Bank of Santa Fe
1. D made verbal guarantee of payments for shipments. P produced testimony of the
out of court statements by Ds vice president that they would pay.
2. Oral agreement (k law), thus the credibility of the testifying witness is the issue,
not the credibility of Ds vice president when he made the alleged statements and this
testimony is not hearsay.
4. The oral statements of Philip Levitt were expressly offered for a nonhearsay
purpose. The relevance of Levitts statements is not their truth or falsity, rather it is
the fact that the statements were made. The relevance of the statement depends,
therefore, not on the credibility of the out-of-court declarant, Philipp Levitt, but on
that of the testifying witness.
F. Assertions as to declarants state of mind (Circumstantial Evidence of State of Mind)
1. A direct out of court assertion by the D as to their state of mind is hearsay - because
the fact finder must determine the truth of the assertion in order to determine the
declarants state of mind. If the statement was a joke, it would have no probative
value. If the statement is not used to prove the truth of the matter asserted, however, it
would be admissible.
2. Statements to show state of mind of declarant- Fun-Damental Too v. Gemmy
Industries
o a. Copyright toilet bank manufacturer case. At trial, Ps sales manager testified
that some retail customers complained because they thought that P was selling
the toilet bank to other retailers at a lower price.
o b. The testimony of Ps sales manager was not offered to prove that P was
actually selling to some retailers at lower prices. Instead, it was probative of
the declarants confusion about the product.
o c. The evidence is admissible because it wasnt offered to prove the matter
asserted which was Fundamental was selling their products to other customers
for a lower price. It was to show that there was actual confusion by the
customers.
o d. State of mind of listener and Notice and knowledge of fact or condition
are non-hearsay reasons to give evidence.
o e. The court says the evidence shows the state of mind. By showing the state
of mind you are showing one of the elements of the action.
3. State of mind must be in issue. US v. Hernandez
o a. Drug bust. At trial, the prosecution elicited testimony, over the defendants
objection, from the DEA agent, that an investigation had been opened as a
result of a tip from another federal agency that defendant was a drug
smuggler.
o b. The prosecution offered the evidence to show the agents state of mind and
explain why D was under DEA investigation. However, the agents state of
mind was not at issue. Therefore, the evidence was hearsay.
4. Verbal Conduct to show belief (Implied Assertions)- US v. Zenni
o 1. Bookie Case, agents answered calls placing bets. The US (P) offers this
evidence to show the callers believed the premises were used in betting
operations. In the instant case, the utterances of the absent declarants are not
offered for the truth of the words, and the mere fact that the words were
uttered has no relevance of itself. Rather they are offered to show declarants
belief in a fact sought to be proved.
o 2. Implied Assertions are not hearsay. FRE 801 modified the common law by
defining hearsay as a statement, and then defining a statement as words or
conduct intended as an assertion. This effectively makes nonhearsay all
evidence of conduct, verbal or nonverbal that is not intended as an assertion.
Nothing is an assertion unless intended to be one.
i. Policy: Declarants sincerity is not at issue because a person is acting
in a way consistent with a belief although not intending by his act to
communicate that belief. Self-verifying.
o 3. The utterance of the gamblers was nonassertive verbal conduct, offered for
an implied assertion that bets could be placed at the premises called. Caller
did not intend to make an assertion when they spoke, therefore admissible.
G. Conduct and Circumstance (Nonassertive conduct)
1. Evidence of out of court conduct intended to assert something as the equivalent of
words is treated as hearsay if the words would be hearsay. However, nonassertive
conduct is treated as implied assertions under the FRE and is therefore not hearsay.
2. Suicide as evidence of state of mind- Commonwealth v. Knapp
o a. D was tried for murder on the basis that he aided and abetted Crowinshield,
who was the killer. Crowninshield had committed suicide prior to his trial.
o b. Probative value of suicide: no secret deed goes undetected; man cannot
hide from God.
3. Silence as an assertion- Wilson v. Clancy
o a. P claimed D (atty) defeated a doctors intention to leave half of the value of
his property to P. P offered an affidavit from doctors bookkeeper. The
bookkeeper claims that neither ever mentioned to her the need to change the
titling of the assets. P argues this testimony leads to the inference that such
advice was never given to doctor or D.
o b. Silence, at least where there is no showing of intentional silence as an
assertion, is no longer considered hearsay.
o c. Nonetheless, although the evidence is not hearsay, it should still be
excluded. The probative value of silence, unless under circumstances that
compel speech is weak and so fraught with speculation as to its reason that it
is far outweighed by the prejudicial effect of introducing such evidence.
o d. The hearsay issue in this case is whether silence is excluded by the hearsay
rule. The court looks at the fact that there is several reasons why the Doctor
did not tell Bouman. The inference to be drawn from the silence is not clear.
o e. Under 403, the prejudicial value outweighs the probative value, and in this
case, there are different reasons for the silence. So although it is not
considered hearsay, it cannot be admitted because there could be so many
different reasons for the silence and the jury can only speculate why the
Doctor was silent.
4. Documents to show character of place where found- US v. Jaramillo-Suarez
o a. D was accused of cocaine and conspiracy offenses. The evidence against
him include a pay/owe sheet that recorded drug transactions that was found in
an apartment frequented by D.
o b. The sheet was admitted for the limited purpose of showing the character
and use of the apartment where the sheet was found and not for the truth of the
matter asserted in the document, therefore admissible.
o c. If he signed the pay/owe sheet, this changes everything. It would be more
direct evidence of his drug dealing. It would be party-opponent admission.
5. Declarants belief- US v. Rhodes
o a. Sgt Rhodes (D) was tried for spying. Written information from Soviet
agents was introduced at trial. The information describes his personal
background and that he was working for the Soviets. Purpose was to show
their belief that he worked for them.
o b. Admissibility depends on the framing. If it was introduced to prove he was
spy, then it should have been rejected as hearsay. However, if it was offered to
prove the Soviet Agents THOUGHT he was a spy, then it probably is
admissible.
o c. The information was biographical, and it was admitted because of
McCarthyism. It was used as circumstantial evidence as a fact of
consequence.
6. Knowledge based on statements of out of court declarants- US v. Brown.
o a. Brown (D), an income tax preparer, was charged with preparation and
presentation of fraudulent tax returns. At trial, an IRS agent testified that
between 90-95% of the returns she audited that had been prepared by D
contained substantially overstated itemized deductions.
o b. Agents testimony was based on out of court statements (talking to the
taxpayers themselves), which deprived D of the opportunity to test the
assumptions through cross. The fact finder could not examine the
trustworthiness of Agents testimony because it could not examine the out of
court statements on which it was based. Her testimony, being founded on
hearsay, should have been excluded. (Hearsay w/in hearsay)
H. Testimony by scientific instruments are generally not hearsay. Such instruments lack a
conscious motivation to tell falsehoods and operation of machines can be investigated in court
through human witness. City of Webster Grove v. Quick
1. Quick (D) was convicted of speeding. Conviction obtained on the strength of an
officers testimony concerning an electronic timing device that showed D was
speeding.
2. Evidence is hearsay when its probative force depends, in whole or in part, on the
competency and credibility of some person other than the witness by whom it is
sought to be produced. The hearsay rule does not apply to what the witness on the
stand observed either through the use of his own senses or through scientific
instruments. Otherwise, a doctor would not be allowed to testify as to results heard
through a stethoscope.
3. The officer himself testified to the reading of the mechanism in question and not to
what someone else had told him; thus, the hearsay rule doesnt apply. The witness
when testifying was under oath, and was thoroughly cross-examined, thus satisfying
the principal requirements of the hearsay rule. The hearsay rule cannot be applied to
what the witness, on the stand and subject to cross-examination, observed, either
through his own senses or through the use of scientific instruments.

Not-Hearsay
I. FRE 801 (d) (1) in depth
The following definitions apply under this article:
(d) Statements which are not hearsay.
o A statement is not hearsay if--
(1) Prior statement by witness. The declarant testifies at the trial or
hearing and is subject to cross-examination concerning the statement,
and the statement is (A) inconsistent with the declarant's testimony,
and was given under oath subject to the penalty of perjury at a trial,
hearing, or other proceeding, or in a deposition, or (B) consistent with
the declarant's testimony and is offered to rebut an express or implied
charge against the declarant of recent fabrication or improper
influence or motive, or (C) one of identification of a person made after
perceiving the person; or
The rule requires in each instance, as a general safeguard, that
the declarant actually testify as a witness, and it then
enumerates three situations in which the statement is excepted
from the category of hearsay.
(a) Prior inconsistent statements traditionally have been
admissible to impeach but not as substantive evidence. But
under this rule they can be used as substantive evidence.
The trier of fact has the declarant before it and can
observe his demeanor and the nature of his testimony as
he denies or tries to explain away the inconsistency.
Hence, it is in as good a position to determine the truth
or falsity of the prior statement as it is to determine the
truth or falsity of the inconsistent testimony given in
court.
(b) Prior consistent statements traditionally have been
admissible to rebut charges of recent fabrication or improper
influence or motive but not as substantive evidence. Under the
rule they are substantive evidence.
The prior consistent statement must have been made
before there was a motivation to lie. When did the
motive to fabricate or lie occur and did the prior
statement predate this? If it doesnt it cant come in for
the truth of the matter asserted.
(c) basis is the generally unsatisfactory and inconclusive nature
of courtroom identifications as compared with those made at an
earlier time under less suggestive conditions.
What if they have a failure of recollection? Can
someone else testify as to the identification? If the
declarant made the identification and a police officer
was present to the identification, can the police officer
present at the time of the identification that way rather
than the out of court witness herself? Yes because the
out of court statement is being testified to with personal
knowledge of the statement that was made. The police
officer can testify that the statement was made, but the
person must be available to be cross-examined.
II. FRE 801 (d) (1) In depth
A. Prior Inconsistent Statements: 801(d)(1)(A)
1. Elements:
o a) declarant must be a witness subject to cross
o b) prior statement must be inconsistent with current testimony in court
o c) prior statement must have been made under penalty of perjury made at a
trial, at a hearing, or other proceeding or deposition
o If it fulfills these elements the statements can be used to impeach and to the
truth of the matter asserted, but can only be used to impeach if it doesnt meet
C.
2. Foundation
o a. Traditional Rule- Requires the witness be asked about the inconsistent
statement before it is introduced i.e. must ask witness whether he made the
statement (giving it substance), and must name the time, place, and person to
whom it was made. If in writing, must also provide a copy.
o b. Modern rule- it is sufficient if the witness was given an opportunity to
explain or deny the alleged inconsistent statement. (FRE 613 (b))
3. Evidentiary effect of statement
o a. Majority view- cannot be used as proof of facts contained in the statement
(unless hearsay exception) and use is limited to impeach a witness and a jury
must be so instructed.
o b. FRE adopts majority except for one exception: prior inconsistent statements
made by the witness while testifying under oath at some prior trial, hearing, or
proceeding are admissible as NONHEARSAY. As such, they can be used as
substantive proof of whatever was stated.
4. Application of Traditional Rule- Coles v. Harsch
o a. Wife-wrestler case. D called witness to vouch for him. Later P testified that
witness once told him that D's behavior was disgraceful.
o B. The purpose for laying a foundation is to afford all witnesses ample
opportunity to recall a fact before they may be assailed as dishonest.
Therefore, a witness may be impeached by prior inconsistent statements, but
before this can be done, the statements must be related to him, with the
circumstances of times, places, and persons present; and that he has made
such statements, and if so, allowed to explain them.
B. Prior Consistent Statements: 801(d)(1)(B)
1. Elements:
o a) Prior statement must be consistent with declarants current testimony
o b) Other side has to have made an express or implied charge that the statement
in current trial is fabricated
o c) Prior consistent statement had to be made before you had the motive to
fabricate (when did improper motivation allegedly occur? This must pre-
date that)
o Can be used to show truth of the matter asserted
2. Can be used to rebut an express or implied charge against the declarant of recent
fabrication or improper influence or motive. Can be used as nonhearsay substantive
evidence if it falls under this exception.
3. Timing of Prior Consistent Statements Tome v. US
o a. Mother contacted authorities with allegations of sexual abuse by the father
while the child was in his custody. Govt relied on six witnesses regarding out
of court statements made by the child describing the assaults. Statements were
made after the alleged fabrication arose.
o b. FRE incorporates the temporal requirements of common law: Prior
consistent statements are only admissible if made before the alleged
fabrication, influence or motive came into being. These statements were made
after the allegation, therefore was not admissible.
C. Prior Identification: 801(d)(1)(C)
1. Elements
o a) Made by someone at trial now
o b) Was made to ID someone
o Can be used for the truth of the matter asserted.
2. Extrajudicial ids that may or may not be confirmable at trial. Prior ID is usually
admissible because an ID at the time of the event is more reliable in terms of
perception and memory than a later ID. Prior ID is admissible only if the declarant is
available at trial to be crossed, otherwise it falls into the former testimony exception
to the hearsay rule.
3. Identifying witnesss loss of memory- US v. Owens
o a. Victim suffered severe memory loss from his injuries, and when
interviewed by an investigator concerning the assault, could not recall the
name of his attacker. In a subsequent meeting with the investigator, however,
he named Respondent as the perpetrator, and identified Respondent from a
series of photographs. Respondent could not remember visitors, or whether
someone suggested it was D.
o b. D has a right to an opportunity for effective cross (6th Amend), but not cross
that is as effective as D wishes. A witnesss bad memory is one fact a cross
tries to bring out.
o c. FRE 801 (D)(1)(C) means placing the witness on the stand, under oath, and
eliciting willing answers to questions. Memory loss is not an exception to this
rule.
o d. The SC said it was admissible because he is in court and able to cross-
examine about the identification. Similarly, the House Report on the Rule
noted that since, as time goes by, a witness memory will fade and his
identification will become less reliable, minimizing the barriers to admission
of more contemporaneous identification is fairer to defendants and prevents
cases falling through because the witness can no longer recall the identity of
the person committing the crime.
o If the police officer was there to testify to the identification but victim was in
the hospital and could not be cross-examined, this would be hearsay.

III. FRE 801 (d) (2)


d) Statements which are not hearsay.
A statement is not hearsay if--
o (2) Admission by party opponent. The statement is offered against a party and
is
(A) the party's own statement, in either an individual or a
representative capacity or
If witness has a representative capacity and the statement is
offered against him in that capacity, no inquiry whether he was
acting in the representative capacity in making the statement is
required; the statement need only be relevant to represent
affairs.
(B) a statement of which the party has manifested an adoption or belief
in its truth, or
Adoptive admission- Adoption or acquiescence may be
manifested in any appropriate manner. When silence is relied
upon, the theory is that the person would, under the
circumstances, protest the statement made in his presence, if
untrue.
(C) a statement by a person authorized by the party to make a
statement concerning the subject, or
Includes statements to 3rd parties, and statements from agent to
principal.
(D) a statement by the party's agent or servant concerning a matter
within the scope of the agency or employment, made during the
existence of the relationship, or
The tradition has been to test the admissibility of statements by
agents, as admissions, by applying the usual test of agency.
Was the admission made by the agent acting in the scope of his
employment?
(E) a statement by a coconspirator of a party during the course and in
furtherance of the conspiracy.
The contents of the statement shall be considered but are not alone
sufficient to establish the declarant's authority under subdivision (C),
the agency or employment relationship and scope thereof under
subdivision (D), or the existence of the conspiracy and the
participation therein of the declarant and the party against whom the
statement is offered under subdivision (E).
Need more evidence than just the statements to prove the (c),
(d), or (e) exists.
Excluded from the category of hearsay on the theory that their admissibility in
evidence is the result of the adversary system rather than satisfaction of the conditions
of the hearsay rule.
Related to but not as established as principal/agent. The agency theory of conspiracy
is at best a fiction and ought not to serve as a basis for admissibility beyond that
already established.

IV. FRE 801 (d) (2) in depth- impeach and substantive


A. Admissions by Party-Opponent: 801(d)(2)(A)- Any out of court words or acts of a
party that are inconsistent with the position that the party takes in the current proceeding may be
offered to show the truth of the matters asserted in the out of court statements.
1. Sabinas Definition: words or actions inconsistent with the opposing partys
position at trial relevant to the substantive issues in the case offered against the party.
B. General Requirements
1. Unavailability not required.
2. Do not need to have personal knowledge of the facts admitted. Reed v. McCord
o a. D in a medical negligence case testified as to some of the circumstances
surrounding the accident. The coroner then got on the stand and testified as to
some of the things that the defendant had said, over objection by the
defendant.
o b. Admissions do not have to be based on first-hand knowledge to be
admissible. It is highly improbable that a party will say or admit anything
against himself or his own interests unless its true.
C. Adoptive Admissions: 801(d)(2)(B) When a party is shown to have heard and had
full knowledge of the content of a statement made by another person, and the party by
affirmative words or acts shows that they agree with or accepts as true the facts alleged in the
statement, it can be introduced as admission against the party.
1. Admission by silence- If a party is show to have heard and understood a statement
made by another in his presence, and a reasonable person in the partys position
would have denied the statement, then the partys non-denial (conduct) may be
introduced to show he agreed with or accepted as true the facts stated in the other
statement.
o a. US v. Hoosier- Ds girlfriend described sacks of money in their hotel room.
i. D manifested his adoption in the truth of statements of which his gf
by acquiescing in her statements under circumstances which, if untrue,
D would normally deny. The inference is a weak one, however.
ii. Usually, more is required than mere presence of the party against
whom the admission is sought and mere silence.
iii. Here, Rogers had been previously told of the plan to rob a bank.
That fact represents the more needed.
iv. Here they say the evidence can be admitted because they look at the
totality of the circumstances and that there are instances where silence
shouldnt be used for the truth of the matter asserted, but here the
defendant not only talked about robbing the bank prior, but he did not
refuse the statements by the girlfriend.
v. The key here is that it is not just the silence that controls, but that the
circumstances would compel a response. That and the statements of
the plan to rob the bank shows truth of the matter asserted.
2. Judge needs to find an intent to adopt in preliminary hearing- State v. Carlson
o a. D arrested, officer say track marks on arm. Claimed that the tracks were the
result of injuries received while working on a car. The defendant's wife then
started yelling that defendant got the marks from shooting up in the bedroom.
The defendant sat in silence and just shook his head.
o b. It is a precondition that that the party intended to adopt, agree with, or
approve of the contents of the statement of another.
i. Intent is a preliminary question of fact for the trial judge because the
trial judge has the duty to make preliminary determinations regarding
the admissibility of evidence.
o c. A shaking head was too ambiguous to reasonably be deemed sufficient to
establish any particular interpretation.
D. Authorization: 801(d)(2)(C)
1. Masters approval required. Big Mack v. Dickerson
o a. One man left his truck unattended, and it rolled forward crushing the
another man between the two trucks. Both worked for D. The deceaseds
family sued for damages. Driver said he had been having trouble with the
brakes.
o b. The statements must fall under a hearsay exception to be admissible against
D. To qualify as admissions, they must have been authorized by D. There is a
rebuttable presumption of lack of authority when the agent speaks to someone
other than the master w/o the masters approval. This applies for both express
and implied authority, which driver did not have here.
o c. The issues were if Ledays admissions were admissions of the party. They
are not admissions of the party because they are not authorized admissions by
the employer. The statements to Stiles then were hearsay. Statement of Leday
to the VP was an internal admission and not an admission made publicly. In
common law being used by Texas, the court said there was no authority to
speak to the public and there is also no authority given to Leday to talk to
Stiles about the subject.
o d. The court believes Leday did not have authorization to talk to Stiles about
the accident.
o e. In regards to the statement Leday made to the police officer, it was not a
vicarious admission because the police was not investigating for the company
and that means the statements could not be party opponent admissions. There
was no implicit authority for Leday to talk.
E. Admission of an agent or employee can be imputed to and admissible to the principal
or employer if agent is within the scope of employment: 801(d)(2)(D)
1. Mahlandt v. Wild Candid Survival
o a. Boy attacked by wolf in Ds employees backyard. Employee made a verbal
and written admittance to D.
o b. A statement is not hearsay if the statement is offered against a party and a
statement by the partys servant concerning a matter within the scope of the
agency or employment, made during the existence of the relationship.
i. The employee did have actual physical custody of the wolf, and his
opinions, conclusions were adopted by the Board of Directors as the
basis for action by his principal. Once agency, and making the
statement while the relationship continues, are established, the
statement is exempt from the hearsay rule so long as it relates to a
matter within the scope of the agency.
ii. The agent need not have personal knowledge of the event.
iii. In regards to the minutes, they were admissible against Wild Canid
because there is nothing in the rules that say they cannot use it. But it
cant be used against Poos.
2. Application to nonemployees- Sabel v. Mead Johnson
o a. P experienced priapism from Ds drugs. Sought to introduce tapes of
meetings that D had with outside medical experts.
o b. An agency relationship includes
(i) The agents power to alter the legal relationships b/w the principal
and 3rd parties.
(ii) the existence of a fiduciary relationship toward the principal
regarding the matters w/in the scope of the agency.
(iii) the principals right to control the agents conduct regarding
matters w/in the scope of agency.
Of these characteristics, the issue of control normally determines
whether an agency relationship exists.
o c. D didnt have control or authorized the experts. Meeting was a simple
brainstorming session designed to generate ideas for Ds consideration, and
not to establish an official position. So evidence would not have been
admissible.
F. Admissions by Co-conspirators: 801(d)(2)(E)
1. Requirements
o a. The conspiracy itself is established prima facie by independent evidence
o b. Statement itself was made during the conspiracy (before the crime was
consummated or the declarant withdrew from the conspiracy)
o c. The statement was made in furtherance of the conspiracy.
2. Rationale- US v. DiDomenico
o a. Conspirators are each others agents and principles- an admission by one is
an admission by all, and can be used against all, if the statements are made
w/in the scope of agency.
o b. This dependence on agency principle requires that the rule be limited to a
specific conspiracy. Thus, a conspiracy and a conspiracy to conceal an earlier
completed conspiracy are two different conspiracies. Statements made in
furtherance of the 2nd conspiracy cannot be used to demonstrate participation
in the first conspiracy.
3. Co-conspirator statements made before the defendant joined the conspiracy- US v.
Goldberg
o a. At trial, the court admitted against D as an admission the out of court
statements by two coconspirators before he joined the felony
o b. A late joining conspirator takes the conspiracy as he finds it. Evidence was
admissible.
o c. Baines expressed the traditional notion that- insofar as hearsay is
concerned, a late joining conspirator takes the conspiracy as he finds it: a
conspiracy is like a train, and when a party steps aboard, he is part of the
crew, and assumes conspirators responsibility for the existing freight.
4. Actual furtherance required- US v. Doerr
o a. At Ds trial, the court admitted over Ds objection the statements by two
alleged co-conspirators that were ambiguous and vague. (red curtain cant
believe [he] doesnt know whats going on)
o b. The limitation on admission of co-conspirator statements requires more
than superfluous casual conversations. There must be some reasonable basis
for concluding that the statement furthered the conspiracy.
o c. The government says the statements were in furtherance because Mayer
wanted to invest in the clubs. As to the second statement, because the
government contends it as a description of what is going on in the club, they
say it is in furtherance. But the court disagrees with the government, saying
the first statement was a narration of events and the second statement wasnt
part of the information flow because he was mocking his brothers ignorance.
5. Use of co-conspirators statement to prove conspiracy - Bourjaily v. US
o a. Cocaine bust. At trial, telephone statements by Lonardo (seller) about D
were admitted to prove a conspiracy.
o b. Under FRE 104(a) a court is not bound by the rules of evidence, except
privileges, when it makes preliminary factual determinations regarding the
admissibility of evidence. The rule permits the trial court to consider any
evidence whatsoever, subject to privilege.
o c. Co-conspirator evidence cannot alone prove the conspiracy existed; you
need other independent evidence. The statements themselves may be credible
proof of conspiracy, especially when corroborated by independent evidence.
Because the telephone conversation was corroborated by the events in the
parking lot, the evidence was admitted.
Part 2
Hearsay Exceptions
October 22nd
FRE 803
I. FRE 803 Hearsay Exceptions; Availability of Declarant Immaterial
The following are not excluded by the hearsay rule, even though the declarant is available as a
witness:
The present rule proceeds upon the theory that under appropriate circumstances a hearsay
statement may possess circumstantial guarantees of trustworthiness sufficient to justify
nonproduction of the declarant in person at the trial even though he may be available.
(1)Present sense impression. A statement describing or explaining an event or condition
made while the declarant was perceiving the event or condition, or immediately
thereafter.
o The underlying theory of Exception (1) is that substantial contemporaneity of
event and statement negative the likelihood of deliberate of conscious
misrepresentation.
o (1) recognizes that in many, if not most, instances precise contemporaneity is not
possible, and hence a slight lapse is allowable.
o Permissible subject matter of the statement is limited under Exception (1) to
description or explanation of the event or condition, the assumption being that
spontaneity, in the absence of a startling event, may extend no farther.
(2) Excited utterance. A statement relating to a startling event or condition made while
the declarant was under the stress of excitement caused by the event or condition.
o Theory: circumstances may produce a condition of excitement which temporarily
stills the capacity of reflection and produces utterances free of conscious
fabrication.
o (2) the standard of measurement is the duration of the state of excitement.
o The statement need only "relate" to the startling event or condition, thus affording
a broader scope of subject matter coverage.
(3) Then existing mental, emotional, or physical condition. A statement of the declarant's
then existing state of mind, emotion, sensation, or physical condition (such as intent,
plan, motive, design, mental feeling, pain, and bodily health), but not including a
statement of memory or belief to prove the fact remembered or believed unless it relates
to the execution, revocation, identification, or terms of declarant's will.
o The exclusion of "statements of memory or belief to prove the fact remembered or
believed" is necessary to avoid the virtual destruction of the hearsay rule which
would otherwise result from allowing state of mind, provable by a hearsay
statement, to serve as the basis for an inference of the happening of the event
which produced the state of mind)
(5) Recorded recollection. A memorandum or record concerning a matter about which a
witness once had knowledge but now has insufficient recollection to enable the witness to
testify fully and accurately, shown to have been made or adopted by the witness when the
matter was fresh in the witness' memory and to reflect that knowledge correctly. If
admitted, the memorandum or record may be read into evidence but may not itself be
received as an exhibit unless offered by an adverse party.
(6) Records of regularly conducted activity. A memorandum, report, record, or data
compilation, in any form, of acts, events, conditions, opinions, or diagnoses, made at or
near the time by, or from information transmitted by, a person with knowledge, if kept in
the course of a regularly conducted business activity, and if it was the regular practice of
that business activity to make the memorandum, report, record or data compilation, all as
shown by the testimony of the custodian or other qualified witness, or by certification
that complies with Rule 902(11), Rule 902(12), or a statute permitting certification,
unless the source of information or the method or circumstances of preparation indicate
lack of trustworthiness. The term "business" as used in this paragraph includes business,
institution, association, profession, occupation, and calling of every kind, whether or not
conducted for profit.
(7) Absence of entry in records kept in accordance with the provisions of paragraph (6).
Evidence that a matter is not included in the memoranda reports, records, or data
compilations, in any form, kept in accordance with the provisions of paragraph (6), to
prove the nonoccurrence or nonexistence of the matter, if the matter was of a kind of
which a memorandum, report, record, or data compilation was regularly made and
preserved, unless the sources of information or other circumstances indicate lack of
trustworthiness.
(8) Public records and reports. Records, reports, statements, or data compilations, in any
form, of public offices or agencies, setting forth (A) the activities of the office or agency,
or (B) matters observed pursuant to duty imposed by law as to which matters there was a
duty to report, excluding, however, in criminal cases matters observed by police officers
and other law enforcement personnel, or (C) in civil actions and proceedings and against
the Government in criminal cases, factual findings resulting from an investigation made
pursuant to authority granted by law, unless the sources of information or other
circumstances indicate lack of trustworthiness.

III. FRE 803 in depth- Hearsay Exceptions


A. FRE 803 (1) Present Sense Impression
1. Requirements for admissibility- State v. Jones- Police sexual assault case, victim was
chasing officer- separate trooper testified that he overheard a CB radio transmission
between two speakers to the effect that a Smokey Bear," was driving fast with no lights
on and a little car was trying to catch up with him.
o a. No requirement that declarant be available or identified.
o b. Declaration must be made while the observer was engaged in the conduct or
perceiving the event that the declaration was offered to explain.
o c. The declarant must have personal knowledge.
i. BUT no need for the accuracy of the declaration to be corroborated by
the testimony of an equally percipient witness (although it would enhance
testimony)
o Do you need personal knowledge for present sense impression? Jones says there
is two issues: Party offering the statement must show that the truck drivers were
speaking from personal knowledge. You can prove personal knowledge and
contemporaneity by the way the declarants described the way they saw the events;
they were talking in present tense. You can show personal knowledge by the
statement itself, or you can use extrinsic evidence.
o What about corroboration? The court says you dont need to corroborate the
statement with extrinsic evidence. The corroboration is there in this case because
of the statements by the truck drives. You dont need corroboration per se, but in
many cases corroboration exists anyway.
o Do we know who the truck drivers are? No- they are unknown. Even if the
identity of the declarant is unknown, the evidence can be admitted for present
sense impression based on the elements of the rule. There is no requirement of
corroboration, but there is an element of personal knowledge and
contemporaneity. These can be proved in the case.

B. FRE 803 (2) Excited Utterance


1. Requirements
o a. Startling event. The occurrence producing the exclamation must be startling
enough to produce shock, excitement, or a similar reaction in the observer.
i. Occurrence of the event can be shown by the statement itself if the
surrounding facts and circumstances impart a reasonable measure of
corroboration.
Truck Insurance v. Michling- P died after hitting his head at work, no
one saw. Not admissible because the narrative itself is being used to prove
the circumstances from which it is said to derive its credit.
The court did not allow the statement because the statement could not be
proven by anything else other than the statement itself. You cant prove the
startling event by the statement itself.
This is just an outlier and we rely on the Jones rule.
State v. Jones Sub-Issue: What about excited utterance? Can we say an
excited utterance occurred in this case? There is evidence of Officer
Kinesman meeting with the complainant on the road. She said she was
assaulted by a cop and was agitated. The statement describes the
complainant agitated and distraught, so there is evidence the statement
was an excited utterance. A sexual assault is a startling event, and she
showed that she was still under the emotional impact of the assault. Time
is another factor. The statement was made about half an hour after the
startling event occurred, which is an issue because there may be time to
fabricate. But because she was visibly shaken by the event, and this was
the first time she could tell someone of the event, this is admissible as an
excited utterance.
o b. Spontaneity. The statement must be spontaneous, made while under the
influence of the shock or excitement. It must be made contemporaneously with
the event; or shortly after.
o c. Scope of Statement. The exclamation must relate to the exciting evept that
provoked it. Many courts will not allow statements of opinion, particularly when
thy fix blame
2. Statement cannot be an opinion
o a. Lira v. Albert Einstein Med Ctr- P wanted to introduce doctors statement,
Who is the butcher that did this to you? Not admissible because Dr. was a
specialist and could not be shocked by the abnormal condition. More of an
opinion based on medical expertise.
The factor that drove the court to decide the evidence was inadmissible
was that he was a medical professional. The court says the statement is not
an excited utterance because it is a doctor used to seeing these conditions
and thus not startling. The court wants the case to be decided on expert
testimony and not on an utterance.
3. Lira Factors
o a) Nature of the event itself
o b) Nature, behavior and condition of the declarant
o c) Content of the statement
o d) Declarants age and profession
o e) Physical Proximity or psychological perspective of the event.
o f) Statement is unsolicited in response to a question- Evidence admitted in
response to a question is tricky because you have time to reflect before you
answer. It diminishes the spontaneity of the statement.

C. FRE 803 (3) State of Mind


1. When the state of mind is at issue, declarations as to state of mind at the time in
question are admissible, provided they are made under circumstances indicating sincerity.
o a. Rationale- Such declarations have some degree of spontaneity and may be the
best evidence of the declarants state of mind. Because the declarant knows their
own state of mind there is no need to check perception, and because it is a
declarants present state of mind, there is no need to check memory.

FRE 804(b)(2) Statement Under belief of imminent death


Threshold requirement- The declarant is unavailable. Declarant must has to believe they
are in imminent danger of death.
804(B)(2)- Statement under belief of imminent death- Under what cases would the
exception apply to? It would apply to both. There is a limitation for the use of the
evidence in criminal case because a person has a right to confront the witness. There is a
suspicion or belief that the evidence is not trustworthy for someone who is about to die. A
person can make any statement about their impending death, including revenge. When
the FRE was created, this was considered. It can be used in civil cases, but if you have
criminal cases, you must have a homicide.
The history is that people tend to be honest to atone for their sins. It has an origin in
religious beliefs. That is still the reason that the rule exists. There are short segments in
the case book of different ways to view someone being close to death and aware of it.

State v. Jensen (2008)


This case is about a letter the deceased wrote before she was killed. She wrote that she
feared for her life and her husband might kill her. The issue is whether we can offer the
letter she wrote before she died as a dying declaration.
The judge concluded that the statement was a dying declaration because it fulfilled the
imminence requirement because the statement only has effect when the person opens it
and a person reads it. How did the judge say that the letter showed that death was
imminent? Cardozo says there must be a settled, hopeless expectation that death is at
hand. The court said that when she is writing the letter she is swift and impending doom.
She could have withdrawn that letter and the act of not withdrawing showed she had a
continuing fear that harm would fall upon her at the hands of her husband. Historically,
dying declaration is a way to avoid the confrontation clause. Patient must speak with
swift and expecting doom. If it relates to your demise, then it can be used as part of a
dying declaration.
Why dying declaration? A letter is not a statement until it is opened and read. Why is he
using dying declaration as an exception to hearsay evidence? Dying declaration escapes
the confrontation clause because the person is dead. Testimonial evidence which is
brought in court has to be challenged by the defendant if they choose so. Dying
declarations whether testimonial or not, doesnt sue confrontation clause because person
is dead and cannot be cross-examined

2. Statements that go beyond indicating the state of mind of the declarant can still be
admitted, but must be given proper jury instructions. (Adkins v. Brett)
o a. Alienation of spouse. P sought to introduce wifes statement: she had gone
automobile riding with D, had dined with him, had received flowers from him,
that he was able to give her a good time and P was not, that she intended to
continue to accept D's attention and P could do what he pleased about it, and that
he was distasteful to her.
o b. Evidence that is competent for the purpose of showing the state of mind, is not
rendered incompetent by the fact that it also tends to prove other material matters
that it is not competent to prove (flowers, cars, etc).
o c. All that is needed with proper jury instruction, or if other evidence is available
that proves state of mind, that should be used instead of his statement. Can also
excise portions that are not relevant to the state of mind.
o The issue was whether to admit evidence of conversation between plaintiff and
his wife. Wife talked about going out with the defendant and she would continue
to date him. What is also important in the statement is that the plaintiff can do
whatever he pleases and that he was distasteful to her.
In regards to the statement, the last part of it reflects the state of mind of
the husband being unpleasant to her. This reflects alienation. Cause of
evidence is alienation of affections and the question is if the defendant
caused the alienation. We need to focus on the last part of the narrative
because this reflects a certain state of mind. It doesnt answer the question
of if the defendant caused the alienation, or does it?
o You have to instruct the jury that the evidence is used to prove the state of mind
and not the main issue. They are not used for the truth of the matter asserted, but
rather the wifes state of mind vis a vis the husband.
3. Declarations of present intent can also be used to show the probability that the person
committed some subsequent act pursuant to that state of mind. (Mutual Life Insurance
v. Hillmon)
o a. Unidentifiable body. The trial court refused to admit letters from Walters
expressing his intention to go with Hillmon (the husband) to Colorado, where the
accident took place.
o b. The letters could be used to prove intent, and intent is shown, then the intent
can be used to infer that it was likely that declarant acted in accordance with his
intent.
o The court rules that the letters were competent of Walters intention. It was a
material fact in deciding whether the body was Walters or Hillmon. The letters
showed Walters intention to engage in future conduct, which makes it more
probable that you engaged in that future conduct. Hillmon becomes a combination
of exception to hearsay rule and use of relevant evidence. Walters letter reflected
his present intent to engage in future conduct which makes it more likely that he
engaged in that future conduct than would without that evidence. Then comes the
use of whose body was it at Crooked Creek.
o Letters are more trustworthy than memory because you would be questioning his
memory and perception through past events. Letters discuss present state of mind
so they better show sincerity and state of mind in a particular moment.
o The Hillmon doctrine says if you are bringing in evidence to show conduct of a
third party, there must be external evidence to corroborate that the third party
engaged in the conduct in question.
4. Declarations to show past conduct is not admissible, statements must be
contemporaneous or forward looking. (Shephard v. US)
o a. Wife was murdered. Said her husband murdered her. Court said it was not a
dying declaration or admissible under state of mind.
o b. Declarations of intention, casting light upon the future, have been sharply
distinguished from declarations of memory, pointing backwards to the past. The
testimony now questioned faced backwards and not forward. What is even more
important, it spoke to a past act, and more than that, to an act by someone not the
speaker.
o c. To allow state of mind of past actions, is contrary to Hillman and would destroy
the hearsay rule.
o Petitioner Charles Shepard was convicted of killing his wife. He apparently
poisoned his wife. The government admitted evidence of a conversation where
Mrs. Shepard while ill in bed spoke to her nurse and stated that her husband
poisoned her. This was after she asked for a bottle of whiskey. The nurse said the
smell and taste of the whiskey was strange. The nurse testified to the wife making
those statements.
o The governments basis for entering the evidence in the first trial as a dying
declaration, but the wife was on the road to recovery. For dying declarations, you
need evidence of the state of mind of the declarant that death was imminent.
Without that factor, the statement is not a dying declaration. The government tried
to fix it by providing more facts that death was imminent to no avail.
o The defendant introduced declarations of Mrs. Shepard that she was bent upon
suicide. The government again wanted to bring evidence of the conversation
between the nurse and Mrs. Shepard to use state of mind evidence to prosecute
the claim against the defendant.
o The court had a problem with the fact that the government had offered it for the
second the time for the truth of the matter asserted which directly incriminated the
defendant. The problem is Mrs. Shepard made a statement of what had already
happened and was a statement looking back. The evidence is being offered that
she didnt possess a suicidal state of mind, but rather a fear that Dr. Shepard was
trying to kill her.
o The court says the evidence doesnt meet the reach of the exception because her
statement spoke to a past act. The government did not show the declarants
statements to prove her present feeling. We have backward looking statements
which cannot be used for present state of mind. Even if it is admitted, the
evidence involves the state of mind of a third party, and the nature of the evidence
cannot be used properly by the jury. It is not supposed to be used for the truth of
the matter asserted, but the inference that if someone makes a statement about
being poisoned, it is supposed to disprove the accusation of being suicidal.
5. States of mind generally cannot be used to implicate or reflect upon the probable
conduct of a third person. EXCEPT: Intent to do something with another person. (United
States v. Pheaster)
o a. Larry was allegedly kidnapped by Angelo. Larry had made statements to
friends that he intended to go meet Angelo in the parking lot of the Sambos
restaurant, and pick up a free pound of marijuana from him.
o b. The inference to be drawn from a persons intent to perform an act is that he did
that act. If the statement implies that a second person would engage in some
future conduct with the declarant, that makes the likelihood smaller that the
conduct actually took place, but it does not make the actual statement any less
reliable (it is still free from memory lapse).
o c. A statement of a declarant is not admissible solely to show that a 3rd party
acted in conformity with his/her intent. However, a statement of a declarant is
admissible to show the declarant acted in conformity with his/her intent together
with a 3rd party, but the 3rd party is entitled to a limiting instruction.
o You can use the state of mind of the declarant to prove inferentially the other
matters at issue. And the fact that he was meeting a third person doesnt matter
because he was not commentating what the third party was going to do and it just
showed that Larry was meeting someone at the parking lot. The goal of the offer
of the evidence by the government is to place Angelo at the scene of the crime. It
is a powerful inference made in the state of mind exception.
o The question becomes does this mean we could place defendant at the scene of
the crime? You cant use it to show the third partys presence or purpose and you
need independent corroborating evidence to prove Angelo was there. What facts
are there that lend corroboration? There is evidence that Larry was seen with
Angelo. Not the strongest of evidence, but circumstantial evidence will suffice.

D. FRE 803 (5) Past Recorded Recollection


1. If a witness indicates that she has no present memory of the facts that she observed
earlier but that she made a record of those facts, then the record is hearsay when offered
to prove the facts asserted. However, the record may be admissible as an exception to the
hearsay rule if a proper foundation is first laid.
o a. If the witness is allowed to use the written record, she escapes cross-
examination, except as to the making of the record. However, if the requirements
stated below are followed, the record is probably as reliable as the witnesss
testimony, since the possible defect of memory is avoided
2. DISTINGUISH FROM Rule 612. Writing Used to Refresh Memory
o a. A testifying witness may be permitted to refresh or revived her memory by
referring to a writing or anything else if the witness will thereafter be able to
testify from present recollection (i.e without depending on the writing)
o b. Foundation must be laid (look for these in a fact pattern)
i) Before reference to the writing, the witness has some present memory,
however, it is defective
ii) After presentation to the court and opposing counsel, the writing is
shown to the witness and
iii) After reference thereto, the witnesss memory is refresh and they are
able to testify w/o depending on the writing.
o c. Writing need not be authentic, or made by the witness or even have independent
relevancy. Therefore, party using it does not have any right to offer the writing
into evidence.
i) However, the adverse party has the right to inspect the document and to
introduce into evidence any portion that relates to the witness testimony.
o d. Baker v. State- Murder D, trying to show witness report made by another
officer stating that victim said D wasnt the one who attacked him. The use of the
hearsay writing is for purposes of sparking memory. It is not the document that is
going into evidence, but the recollection of the witness.
i) For refreshing memory, unlike recorded no need for personal
knowledge, recording it immediately after event, or witness vouching for
accuracy.
Officer Bolton is on the stand and Bakers attorney wanted to refresh his
recollection with the statement written by Hucke. The judge did not allow
this to happen
Past recollection recorded Pg. 329 says two things must be established- (1)
Record must have been adopted by the person at the time it was made, and
(2) a person can testify that when the record was made he knew it was
accurate.
3. Requirements for 805 (c)
o a. The document was prepared or adopted by the witness.
o b. Such preparation or adoption occurred when the matter was fresh in the
witnesss memory
o c. The document correctly reflects what was remembered when it was made
o d. The witness has insufficient recollection to testify fully and accurately about
the matter; and
o e. The document is the authentic memorandum which has not been tampered with.
4. Adams v. NY central Railroad- D claimed Ps injury antedated the claimed injury. D
trying to introduce an insurance agents written notes as past recollection recorded. P
objected, saying it wasnt routine or contemporaneous. Court sustained.
o a. Memo should have been admitted, trial court confusing recorded w/ business
record exception.

D. FRE 803(6) Business Records Exception


Rationale: Business records should be admissible, despite the hearsay rule, when the
sources of information and the method and time of preparation indicate their
trustworthiness.
o a. Requirement: Entry made in the regular course of business
What is the guarantee of trustworthiness in the business record exception? You are
making the record for business purposes rather than thinking of making the record
because you are going to trial. You are relying on nature of business to maximize profits,
so the records they maintain are reliable and accurate because it furthers the business
interest of the entity in question.
2. Does not encompass hearsay within hearsay. Need to account for each level.
o a. Johnson v. Lutz- D sought to introduce report made by officer at scene.
Contained statements of a witness at the scene. Held: The doctrine is not extended
to include memoranda of statements from people who were not engaged in the
business and not made in the pursuance of any duty owing by the person making
the statement.
The court distinguishes economic purposes to this case where this was not
made during the course of a transaction. The police officer came to the
scene and bystanders told her what happened, and she wrote in the report.
The officer working under the normal course of business because a police
officer at an accident scene is to interview witness and record them. The
bystanders are not acting under any business function; they have no duty
to report. A police officer has duty to report, but bystanders do not have a
duty to report. They are not employed to the police. Because they was no
duty to report, the document was hearsay and not admissible.
o b. See also US v. Duncan infra.- D attempting to defraud insurance co. P sought
to introduce insurance co. records that D had been in the hospital numerous times.
Appellate Court found that although the insurance companies' records contained
information that they did not produce, they received the information from other
businesses (aka the hospitals and doctors). Since the records from the other
businesses were themselves business records, there is no "accumulation of
hearsay", and the entire package is still admissible.
o c. Sana v. Hawaiian Cruises- P sought to introduce insurance investigation that
contained an unsworn, out-of-court statement by investigator, recounting
unsworn, out-of-court statements by Ps co-workers, recalling P s unsworn, out-
of-court statements. Admissible because each layer of hearsay was accounted for.
i. Ps statements were admissible under the exception for statements of the
declarants then-existing physical condition. Ps coworkers were being
interviewed by the companys insurance agency within the scope of their
employment and thus their statements are admissible as admissions by a
party-opponent.
3. Business activity: Definition includes every business, institution, association,
profession, occupation and calling of every kind
4. Regular course of business- To assure trustworthiness, it must appear the records
were prepared in the regular course of business and that it was the regular practice to
make the particular record or entry in question.
o a. Entrant under duty to record- It must appear that the entry was made by
someone whose duty it was to make such entries as part of his employment.
o i. US v. Vigneau- Money Laundering, receipts werent part of the regular course
of business because they were filled out by the Ds and not clerks.
Two brothers were convicted for a drug distribution scheme. The evidence
at issue here was the Western Union To Send Money Forms that were
offered for the truth of the matter asserted for the money laundering
charge. The form is filled on the left hand side with their name and address
and the clerk fills out the right and the money is wired.
The defendant challenges admissibility because there was no evidence of
the identity of the sender of the money. If there was an independent
verification system, the evidence would be admitted as a party opponent
admission. So basically any person could say a false name and no one
would check. When the defendant filled out the forms and reliability of the
forms are in question, the question is that if these are business records that
can be admitted? The impediment to this would be that there is no duty to
report by the person filling out their name (hearsay within hearsay). So
this fails as a business record exception to the rule.
o b. Records related to primary business and customarily maintained by the
organization.
o Hospital Records-
i. US v. Duncan- D attempting to defraud insurance co. P sought to
introduce insurance co. records that D had been in the hospital numerous
times. Appellate Court found that although the insurance companies'
records contained information that they did not produce, they received the
information from other businesses (aka the hospitals and doctors). Since
the records from the other businesses were themselves business records,
there is no "accumulation of hearsay", and the entire package is still
admissible.
The defendants were charged with defrauding insurance companies
by buying hospitalization policies and purposefully getting into
accidents. The record of the insurance companies were in question
and there were three issues.
ii. Williams v. Alexander- D attempting to introduce hospital records
where P stated the reason for his injury was not Ds fault. Inadmissible.
Hospital records are generally held admissible only to the extent that they
are reasonably related to the diagnosis and treatment of patient conditions.
Plaintiff claims that defendant went into intersection and defendant
says he was hit by someone and propelled into the intersection.
Plaintiff brought in evidence of hospital records and the defendant
entered the rest of the medical record which said that the plaintiff
told the physician that the defendants car was hit from behind and
hit him.
The standard the court used is that the statements have to serve a
medical purpose. It has to be germane to the diagnosis or the
treatment. Statements and medical business records, even though
party opponent admissions are not admissible because the
physician had no need to question how the accident occurred. FRE
803(4) shows that the medical statements can be admitted if they
are reasonably pertinent to diagnosis and treatment. Statements
made to physicians or hospital personnel that are not pertinent to
diagnosis and treatment are not admissible. So if you have a
hospital record that the hospital keeps in the normal course of
business, not all the statements will be admissible because many of
the statements are not pertinent to diagnosis and treatment.
Note: Statements about the cause of injury may or may not be so
related (sometimes the cause of the injury is relevant to the
diagnosis!)
Accident Reports- If there is an indication of motive to fabricate records or
if records are self-serving, it is inadmissible.
iii. Palmer v. Hoffman- P Railroad accident victim. D sought to have an
investigation report of the accident introduced, made by engineer involved
in accident. Inadmissible, the report was not made in the systematic
conduct of business. Unlike payrolls, bills, etc, this reports primary
purpose is preparation for litigation.
At the time the trial occurred and the company offered evidence,
he was dead. So we have an unavailable declarant. The evidence is
offered as 803(6) as a business record. The issue with this record is
that the declarant has a huge incentive to say that it is not his fault
(trustworthiness of the record). The company may have made this
record in contemplation of a negligence case.
The court discusses the record vis a vis running a railroad is that
the business is railroading not creating documents in anticipation
of a trial. Routineness to help a business operate as a business, not
to help them prepare for a case.
Records that are untrustworthy based on reason they were
compiled or timing they were compiled makes us consider the
Palmer case.
iv. Lewis v. Baker- Railroad accident. D sought to introduce accident
reports that the brake had worked before. Admissible because the record
made pursuant to a regular procedure at the railroad within a reasonable
time after the accident. P cited Palmer as to inadmissibility. But in that
case, the report had been made by one of the persons involved in the
accident. Here, by contrast, the report was made by persons who were not
directly involved in the accident and who could not have been sued by P.
Train accident where there was a test for the brakes on the car.
Plaintiff was hurtling down the hill and jumped off, which is why
he was hurt. Defendants theory is that the plaintiff made a mistake
in operating the car, panicked and jumped off.
The mere fact that a record could ultimately he used in litigation
does not mean that it must be excluded. There must be a motive to
fabricate on the part of the preparers before the business record
will be excluded.
v. Sana v. Hawaiian Supra. Insurance report not a self-serving
document offered by the party who created the report. Shipowners and
crew had a duty to investigate seamans injuries. Can be seen as routine in
the maritime industry.
Sana is an employee on Hawaiian cruises. He left work and met his
father who noticed he was shivering and had a bump on his head.
Sana stayed at home and slipped into the coma. The illness he had
was viral encephalitis. Sana claims Hawaiian cruises is responsible
for maintenance and cure for his sickness.
The issue is whether this illness begin while he was on the ship in
employment of the company without regard of the company being
responsible for it. He hit his head March 10th and they had a rep
from the insurance company come to investigate the claim.
Rutherford went to three workers of the boat. Rutherford and the
co-workers were not at trial, so the record was trying to be
admitted by the plaintiff.
We conclude from the record that Sanas co-workers were acting
within the scope of employment. Also, Hudson had an independent
duty to monitor and report any injuries or illnesses among his staff.
In all likelihood, Sanas co-workers would have been disciplined
had they refused to share information with their employers
insurance investigator. The fact that the interviews were conducted
on company time supports this view.
Sana claims that Rutherfords information gathering was a
regularly conducted business activity on behalf of Beaudry
Insurance. It was admitted.
Moreover, the report had other circumstantial guarantees of
trustworthiness. Rutherford had a duty to prepare an accurate
report for Beaudry Insurance which presumably relied on that
report to adjust Sanas claim.
5. Contents of Entry
o a. Facts v. Opinion- FRE allows into evidence entries of acts, events, conditions,
opinions, or diagnoses if they were made in the regular course of business.
Applies to Business Records AND Public Records.
o b. Beech Aircraft v. Rainey- Airplane crash. D introduced an investigative report
prepared by a military investigator that was divided into sections labeled finding
of fact, opinions; and recommendations.
i. Held: The Advisory Committee did not distinguish between fact,
opinions, and conclusions; and actually explained that the rule assumes
admissibility of such reports, with the escape provision when there is a
lack of trustworthiness.
ii. As long as a conclusion or opinion is based on a factual investigation
and satisfies the trustworthiness requirement, it is admissible with the rest
of the report. If the judge determines that a report, or portions thereof, is
untrustworthy, it may be excluded.
o c. HOWEVER: US v. Oates- Evaluative and law enforcement reports are
inadmissible against defendants in criminal case
Defendant was convicted of possession of heroin and attempt to distribute.
The reports in question was an official report and a worksheet prepared by
a chemist for US Customs. The chemist was named Weinberg. He was
available for testimony originally but in the later date he was sick. So the
Customs agency brought Harrington to testify and she testified on the
regular practices of chemists of analyzing drugs.
i. Chemist report identifying substance as heroin.
-They do not fall within the business records exception, because
they are not governmental reports setting forth factual findings
resulting from an authorized investigation.
-Cannot qualify as public records and reports as they were
prepared by law enforcement personnel in connection with an
investigation
ii. When a police record is offered against the accused in a criminal case,
the adversarial atmosphere in which the record was created detracts from
the trustworthiness. Congress manifested its concern about the dangers of
police records by prohibiting their use against the accused under the public
records exception. Moreover, because of the trustworthiness dangers, such
records are also inadmissible when offered against the accused under the
business record exception.
The rule at issue are 803(8)(B) and 803(8)(C). The 803(8)(B) was at issue
because the chemist may be part of law enforcement personnel. There is
not a level of trustworthiness because they have a motive to convict him.
Under (B) you cannot offer police reports based on the terms.

E. FRE 803 (7) Business records as absence of any entry


1. Common law
o a. Business entries were permitted only to prove the facts contained therein. It was
not permissible to use a record for negative purposes.
2. Modern view
o a. Properly authenticated business records can be used to prove the occurrence or
nonoccurrence of a transaction if it can be shown it was the regular practice of
the organization to record all such transactions.

F. 803 (8) Public Records Exception


Three Categories of Records and Reports
(a) Activities of the office
o You can name any federal or state agency you want that has requirements to
report the activities of the agency. National Relations Board for example records
complaints issued against employees and unions on a monthly basis.
(b) Matters observed pursuant to duty of law
o Distinction between civil and criminal cases is important for this and C. This
exception ranges from the US Weather Bureau recording how much rainfall falls
on the US during the year to the Police Reports that are recorded and used in
cases. Admissibily is based on what law enforcement personnel means.
(c) Investigative Report
o C is broader than B. So many federal or state agencies investigate events that
occurred and try to find answers to causation, fault, or historical reckoning with
what transpired. You can name any agency you want and show they have an
investigative aspect.
o It can be compiled in many ways. It can be an administrative hearing where an
entity is ordered to make an investigation. Another way is after an accident where
the entity interviews people why an accident occurred. So these reports have
opinions, conclusions, and in medical investigations diagnoses. The issue with
investigatory reports is that conclusions may be based on hearsay evidence. There
is the expertise in an agency can properly screen out what is unreliable for the
ultimate opinions and conclusions. So the agency can use hearsay evidence for the
opinion and conclusion. But where there is a trial where the report is implicated,
the hearsay that may be existent in the report may still be rejected to. Conclusions
are still admissible, but not the
What gives the circumstantial guarantee of trustworthiness? They are created by public
institutions and entities who have a duty imposed by them by law to carry out their
respective interests for the public interest. So this means that anyone in the organizational
structure who is supposed to carry out the mission of the agency is what is required under
law and will carry it as best as possible as employees of the organization. There is overlap
business record exception and public record exception.
1. Rationale: Public officers would frequently have to leave their jobs to appear in court
and testify as to acts done in their official capacity, and because the public officer is under
a duty to report properly, the records are also probably reliable.
2. Requirements
o a. Duty to record- records must be prepared by a public employee acting within
the scope of their duties; i.e there must be a duty to record the facts involved.
i. US v. Grady- Gun sale to Irish rebels. Conviction was based partially on
the records of firearms entries made by Irish police.
-Usually cant use police reports, but these reports, however, were
prepared pursuant to a duty to report the facts. They were strictly
routine records.
- The difference between the records in this case and those in
Oates supra has to do with the subjective nature of the elements
going into the reports. While both reports were routinely prepared,
the analysis required in the chemists report is qualitatively
different from the copying of the weapons serial numbers.
-HOWEVER: Police reports are not usually admissible against a
criminal defendant.
If the issue is the record of photos of license plates that cross the
border in deportation cases of undocumented immigrants. Is this
admissible under 803(8)(B)? Information that is obtained by law
enforcement may not be admissible, but it is a photograph of a
license plate it is a routine function and admissible. Because it was
the routine recording of the license plate numbers of cars crossing
the border at a given time in a given period. If you show a motive
that is different from routine and administerial, then it may not be
admitted.
ii. U.S. v. Oates
The rule at issue are 803(B) and 803(8)(C). The 803(8)(B) was at
issue because the chemist may be part of law enforcement
personnel. There is not a level of trustworthiness because they have
a motive to convict him. Under (B) you cannot offer police reports
based on the terms.
Law enforcement personnel was broadly construed, stating that
anyone in the chain of actors that were part of a prosecution is a
law enforcement personnel. A person from Customs analyzing the
chemicals to see if it was heroin or not was part of the
proseceturial processes and thus that kind of report cannot be
admitted.
C only allows investigative reports in civil cases and only in a
criminal case against the government. It is exculpatory, so if the
defendant chooses to use a report compiled by the government, it
can do so under C. It can be offered in a civil case, and under B,
depending on if the individuals involved were part of the
mechanism involved to seek a conviction, then B is foreclosed as
well.
o b. Personal knowledge of entrant generally required.
i. Must have first hand knowledge of the facts recorded, ad oppose to mere
opinions or conclusions or reports to the official by others.
ii. Exception to personal knowledge requirement: Beech Aircraft Supra.
Factual findings resulting from an investigation made pursuant to
authority granted in law. The report had narrow factual findings and a
broader fact based conclusion of probable cause.

FRE804
IV.Rule 804. Hearsay Exceptions; Declarant Unavailable
a) Definition of unavailability- "Unavailability as a witness" includes situations in which
the declarant
o (1) (Privilege exemption) is exempted by ruling of the court on the ground of
privilege from testifying concerning the subject matter of the declarant's
statement; or
A ruling by the judge is required, which clearly implies that an actual
claim of privilege must be made.
o (2) (Uncooperative witness) persists in refusing to testify concerning the subject
matter of the declarant's statement despite an order of the court to do so; or
o (3) (lack of memory) testifies to a lack of memory of the subject matter of the
declarant's statement; or
The lack of memory must be established by the testimony of the witness
himself, which clearly contemplates his production and subjection to
cross-examination.
o (4) (death or sickness) is unable to be present or to testify at the hearing because
of death or then existing physical or mental illness or infirmity; or
o (5) (cant find witness) is absent from the hearing and the proponent of a
statement has been unable to procure the declarant's attendance (or in the case of a
hearsay exception under subdivision (b)(2), (3), or (4), the declarant's attendance
or testimony) by process or other reasonable means.
o A declarant is not unavailable as a witness if exemption, refusal, claim of lack of
memory, inability, or absence is due to the procurement or wrongdoing of the
proponent of a statement for the purpose of preventing the witness from attending
or testifying
(b) Hearsay exceptions. The following are not excluded by the hearsay rule if the
declarant is unavailable as a witness:
o (1) Former testimony. Testimony given as a witness at another hearing of the
same or a different proceeding, or in a deposition taken in compliance with law in
the course of the same or another proceeding, if the party against whom the
testimony is now offered, or, in a civil action or proceeding, a predecessor in
interest, had an opportunity and similar motive to develop the testimony by direct,
cross, or redirect examination.
Former testimony does not rely upon some set of circumstances to
substitute for oath and cross-examination, since both oath and opportunity
to cross-examine were present in fact. The only missing one of the ideal
conditions for the giving of testimony is the presence of trier and opponent
("demeanor evidence").
Under the exception, the testimony may be offered (1) against the party
against whom it was previously offered or (2) against the party by whom it
was previously offered. In each instance the question resolves itself into
whether fairness allows imposing, upon the party against whom now
offered, the handling of the witness on the earlier occasion
The question remains whether strict identity, or privity, should continue as
a requirement with respect to the party against whom offered. The rule
departs to the extent of allowing substitution of one with the right and
opportunity to develop the testimony with similar motive and interest.
o (2) Statement under belief of impending death. In a prosecution for homicide or
in a civil action or proceeding, a statement made by a declarant while believing
that the declarant's death was imminent, concerning the cause or circumstances of
what the declarant believed to be impending death.
The common law required that the statement be that of the victim, offered
in a prosecution for criminal homicide. Thus declarations by victims in
prosecutions for other crimes, e.g. a declaration by a rape victim who dies
in childbirth, and all declarations in civil cases were outside the scope of
the exception. An occasional statute has removed these restrictions
(3) Statement against interest. A statement that:
o (A) a reasonable person in the declarants position would have made only if
the person believed it to be true because, when made, it was so contrary to the
declarants proprietary or pecuniary interest or had so great a tendency to
invalidate the declarants claim against someone else or to expose the
declarant to civil or criminal liability; and
o (B) is supported by corroborating circumstances that clearly indicate its
trustworthiness, if it is offered in a criminal case as one that tends to expose
the declarant to criminal liability.
The circumstantial guaranty of reliability for declarations against
interest is the assumption that persons do not make statements which
are damaging to themselves unless satisfied for good reason that they
are true.
The requirement of corroboration is included in the rule in order to
effect an accommodation between these competing considerations.
When the statement is offered by the accused by way of exculpation,
the resulting situation is not adapted to control by rulings as to the
weight of the evidence and, hence the provision is cast in terms of a
requirement preliminary to admissibility. Cf. Rule 406(a). The
requirement of corroboration should be construed in such a manner as
to effectuate its purpose of circumventing fabrication.

V. 804 in depth
A. 804 (b) (1) Former Testimony
1. Refers to transcripts of testimony given by a witness at some prior deposition, hearing,
or trial in the same or another case.
2. Requirements for admission: If the party against whom the testimony is now being
offered:
o a. Was a party to the earlier trial or proceeding (or predecessor in interest if a civil
action)
i. Travelers Fire Insurance v. Wright- Ps sued D for proceeds of
insurance policy. D sought to introduce testimony from criminal case of
one of the Ps that he and accomplices intentionally set the fire.
Held: Testimony from criminal cases can be used in civil cases.
And because the co-plaintiff had similar motive and interest to
cross, even though they were not party to criminal case, it is
admissible.
It is a civil case where JB Wright and JC Wright want to recover
under the terms of two fire insurance policies. They are business
partners and the defendant is the insurance company. The
defendant claims the fire was deliberately started by JB with intent
to defraud the insurance company. Prior to this civil case, there
was a criminal case where JB was on trial for the fire related in this
case. He was found guilty and convicted and the evidence at issue
in this civil case is the testimony is the testimony in Eppler and
Brown who were witnesses in the criminal trial. They are now
unavailable to testify in this civil case because they want to use
their 5th amendment rights. The insurance company wants the
transcript from this case to show JB intentionally started this fire.
So the transcripts are going to be offered and the court reporter was
going to come in and testify about the accuracy of the transcript.
As a general proposition we think testimony from a criminal case
can be introduced in a subsequent civil case where it appears that it
is impossible to obtain the testimony of the witness who testified in
the criminal case; that there was an opportunity to cross-examine
the witness by the party against whom the testimony is sought to
be used in the civil case, or by one whose motive and interest in
cross-examining was the same; and that there is an identity of
issues. As will be hereinafter shown, identity of all parties is not an
independent requirement in all cases.
o b. had an opportunity to examine the witness at the time and
o c. had a similar motive to develop the witnesss testimony (by direct or cross) as
that which he now has.
i. US v. Salerno- Mob co-conspirator Ds. Govt sought to introduce
testimony from a grand jury hearing at trial. Held: A prosecutor
questioning a witness before the grand jury is motivated by different goals
than a prosecutor at trial. At the grand jury stage, a prosecutor must try to
discover info w/o revealing the governments hand. At trial, however, must
go all out to impeach a witness.
Involved indictments of RICO violations. These defendants are part of
Construction companies in the 80s. The party at issue was a mafia family.
They allocated contracts within the club and they did this in exchange of
the proceeds.
o d. The issue in both trials do not have to be identical, but they must at least be
substantially the same (they must relate to the same general subject matter as to
assure the same scope of cross)

B. 804 (b) (2) Dying Declaration


1. Allowed in civil actions generally but in no criminal cases, EXCEPT homicide.
Declarant must be unavailable.
2. Requirements
o a. Victim as declarant. The dying declaration must have been made by the victim
and not a 3rd party.
o b. Sense of imminent death. The victim must have made the declaration while
actually believing death was imminent. Most important and difficult to prove
(State v. Jensen- Wife left note that she thought husband was going to kill her.
Cicero: Court is reaching but what you should get out of the case is that there
must be a belief death was imminent)
o c. Scope of declaration. The dying declaration must state facts about the cause or
circumstances of the victims impending death. A mere opinion about the cause of
death does not fall within the exception.
o d. Death requirement. Traditionally, the exception would only apply if the
declarant had actually died by the time the evidence was offered. If the declarant
had a sense of imminent death when the statement was made, it did not matter
how much time elapsed between the declaration and the actual death. Under
Federal Rule 804(b)(2), the victim does not need to be dead at the time of trial if
she is otherwise unavailable and if she believed death was impending When the
declaration was made.
3. Treatment upon admission
o a. Once admitted, a dying declaration is subject to the same attacks as could be
asserted if the declarant were present and testifying. The declaration can be
impeached by proof that the declarant had made prior inconsistent statements or
lacked perceptive capacity. It can also be contradicted by proof of contrary facts.

C. 804 (b) (3) Declarations against Interest


1. When a hearsay declarant not a party to the action has made statements against her
own apparent self-interest, such statements are admitted since they are generally thought
to be trustworthy (and, therefore, cross-examination can be dispensed with).
2. Requirements
o 1. Declarant now unavailable
o 2. Perception and knowledge- The declarant must have had the usual
qualifications of an ordinary witness at the time of the declaration and particular,
personal knowledge of the facts. Generally, opinions or estimates of declarant
are not admissible.
o 3. Against interest- the facts, to the declarants knowledge must be to their
immediate prejudice at the time of the declaration, and the prejudice must be
substantial.
i. GM McKelvey v. General Casualty Co.- P trying to collect on
insurance policy. P submits written confessions of Ds employees. Held:
The prejudice against their interests were substantial because the
confessions were against employees proprietary or pecuniary interest.
Also, there was no motive to falsify.
Employer had fidelity insurance that allowed for recovery in case of theft.
There were written and signed confessions of the theft and the loss, but
these two workers fled town. So did they meet the exception?
4 elements: Thus the rule has arisen that a declaration against interest by
one not a party or in privity with a party to take an action is admissible in
evidence, where (1) a person making such declaration is either dead or
unavailable as a witness due to sickness, insanity, or absence from the
jurisdiction, (2) the declarant had peculiar means of knowing the facts
which he stated, (3) the declaration was against his pecuniary or propriety
interest and (4) he had no probable motive to falsify the facts stated.
They were not available, were gone, it was against their interest to
admit it, and no probable motive to falsify
Traditional statement against interest.
ii. Criminal Liability- under the traditional view, declarations that would
subject the declarant to criminal liability is not admissible. EXCEPT for
an exception under the FRE
iii. EXCEPT: Statements against penal interest may be admissible if
sufficiently corroborated. (US v. Barrett A collection of postage stamps
was stolen. Bucky Barrett was implicated by two witnesses as being
involved. However, there was evidence that one of the persons, now dead,
who had been involved in the crime said that Buzzy, not Bucky was
the person involved. Held: It was against declarants interest because it
implicated him in the crime, but need corroborating evidence to admit the
statement)
As finally enacted, Rule 804(b)(3) requires a two-stage analysis:
first, do the offered remarks come within the hearsay exception as
a statement against interest? and second, if they do, is there
sufficient corroboration to clearly indicate trustworthiness? Here
we believed the remarks offered were statements against interest
within the Rule, and that the district court should have gone on to
determine whether there was sufficient corroboration so as to
warrant their admission.
BUT: Third-party statements that inculpate a defendant are NOT
admissible if they are not inculpatory against the person making them.
(Williamson v. US- Cocaine mule who didnt know about the drugs in the
car refused to testify against the person he was to deliver it to. 804(b)(3)
does not allow admission of non-self-inculpatory statements. This is true
even if they are part of a broader statement that is generally self-
inculpatory. Statements made at an arrest are traditionally suspect; people
being arrested are strongly motivated to implicate others.)
Harris is driving down the road and is pulled over for weaving on
the highway. Harris consents to the search of the car and sheriff
looks in the trunk which had suitcases of cocaine. Harris is
interviewed on the phone by a drug enforcement agent and we get
the first version of the story, which was that Harris got the drugs
from someone in Florida and the cocaine belongs to Williamson.
When Harris is interviewed again, he said that when he received
the cocaine he got a suitcase with instructions. There was lots of
physical evidence tying Williamson to the case.
He implicates Williamson and the prosecution of Williamson is
based on that evidence. The statement of interest is not against
Williamson, but part of it is disserving as to Harris but they wish
use it to prosecute Williamson.
Attorney-Client Privilege
I. Rule 501. General Rule
Except as otherwise required by the Constitution of the United States or provided by Act
of Congress or in rules prescribed by the Supreme Court pursuant to statutory authority,
the privilege of a witness, person, government, State, or political subdivision thereof shall
be governed by the principles of the common law as they may be interpreted by the
courts of the United States in the light of reason and experience. However, in civil actions
and proceedings, with respect to an element of a claim or defense as to which State law
supplies the rule of decision, the privilege of a witness, person, government, State, or
political subdivision thereof shall be determined in accordance with State law.

II. Rule 502. Attorney-Client Privilege and Work Product; Limitations on Waiver
The following provisions apply, in the circumstances set out, to disclosure of a
communication or information covered by the attorney-client privilege or work-product
protection.
o (a) Scope of waiver.
When the disclosure is made in a Federal proceeding or to a Federal office
or agency and waives the attorney-client privilege or work-product
protection, the waiver extends to an undisclosed communication or
information in a Federal or State proceeding only if:
(1) the waiver is intentional;
(2) the disclosed and undisclosed communications or information
concern the same subject matter; and
(3) they ought in fairness to be considered together
o (b) Inadvertent disclosure.
When made in a Federal proceeding or to a Federal office or agency, the
disclosure does not operate as a waiver in a Federal or State proceeding if:
(1) the disclosure is inadvertent;
(2) the holder of the privilege or protection took reasonable steps
to prevent disclosure; and
(3) the holder promptly took reasonable steps to rectify the error,
including (if applicable) following Federal Rule of Civil Procedure
26(b)(5)(B).
o Disclosure Made in a State Proceeding.
When the disclosure is made in a State proceeding and is not the subject of
a State-court order concerning waiver, the disclosure does not operate as a
waiver in a Federal proceeding if the disclosure:
(1) would not be a waiver under this rule if it had been made in a
Federal proceeding; or
(2) is not a waiver under the law of the State where the disclosure
occurred.
(d) Controlling Effect of a Court Order.
A Federal court may order that the privilege or protection is not
waived by disclosure connected with the litigation pending before
the courtin which event the disclosure is also not a waiver in any
other Federal or State proceeding.
(e) Controlling effect of a party agreement.
o An agreement on the effect of disclosure in a Federal
proceeding is binding only on the parties to the agreement,
unless it is incorporated into a court order.
(f) Controlling Effect of This Rule.
Notwithstanding Rules 101 and 1101, this rule applies to State
proceedings and to Federal court-annexed and Federal court-
mandated arbitration proceedings, in the circumstances set out in
the rule. And notwithstanding Rule 501, this rule applies even if
State law provides the rule of decision.
(g) Definitions.
In this rule:
(1) "attorney-client privilege" means the protection that applicable
law provides for confidential attorney-client communications; and
(2) "work-product protection" means the protection that applicable
law provides for tangible material (or its intangible equivalent)
prepared in anticipation of litigation or for trial.

III. 501 and 502 In Depth


A. Basic Rule.
1. A client of an attorney has a privilege to refuse to disclose, and to prevent the attorney
or anyone else from disclosing, confidential communications between the client and the
attorney related to the rendering of legal services. The oldest privilege recognized at
common law, the attorney-client privilege is intended to encourage full disclosure by the
client to the attorney to facilitate proper legal counsel. The privilege belongs to the client;
only the client can waive the privilege. To be protected, the communication must have the
characteristics enumerated below.

B. Attorney.
2. The communication must have been made to a member of the Bar, or to an employee
for transmission to the lawyer. Actual employment of the attorney is not required;
communications are protected even if the attorney does not accept the case or the client
does not hire the attorney.

C. US v. Woodruff- Communication must involve Legal matters


1. Woodruff (D) did not appear for his trial. In order to prove that D knew of the time and
place of trial, and to indict D for bail jumping, the prosecution asked the court to order
D's public defender to say whether he gave the information to D and whether D
understood.
2. The privilege covers communication that relates to a fact the client told his attorney
primarily for the purpose of securing either an opinion on law, legal services, or
assistance in some legal proceeding.
o a. In this case, not legal advice. The attorney has a duty to relay such information
in his role as an officer of the court. Such nonlegal communications are not
privileged, and D's counsel must answer the questions.
o Elements: The privilege applies only if (1) the asserted holder of the privilege is
sought to become a client; (2) the person to whom the communication made (a) is
a member of the bar of a court, or his subordinate and (b) in connection with this
communication is acting as a lawyer; (3) the communication relates to a fact of
which his attorney was informed (a) by his client (b) without presence of
strangers (c) for the purpose of securing primarily either (i) an opinion of law or
(ii) legal services or (iii) assistance in some legal proceeding, and not (d) for the
purpose of committing a crime or tort; and (4) the privilege has been (a) claimed
and (b) not waived by the client.

D. Attorney Work Product and Corporate Clients are protected under privilege (Upjohn v. US)
1. A companys foreign subsidiaries bribed officials and company attorney sent out
questionnaires and conducted interviews. IRS asked for production of these documents.
2. Court broadened attorney client privilege to all employees of corporations acting
within the scope of employment.
3. Documents that reveal the attorneys' mental processes in evaluating the information are
privileged under the work product doctrine.
What is the extent of the privilege in a corporate setting? The court below says that the
privilege does not apply to the extent to the workers. This was termed the control
group- employees of the corporation not making decisions and were not part of the
control group did not have the privilege.
The SC says that any person who provides this information is personifying the
corporation when they are providing information about the corporation itself and doesnt
make sense to limit it to a small group of people.
When mid and lower level employees are making statements in the scope of their
employment it is privileged. Brings back discussion of the issue of vicarious admissions.

E. SF v. Superior Court - Physician's assistance to attorney protected


1. Attorney took his client to Dr. Catton. Catton was not to treat P but only to advise
Hession's attorneys as to his condition so they could prepare their case.
2. Communications from the client to the attorney are privileged. This is true even when
the communication is through an interpreter or through an agent of the attorney.
3. When a communication to an attorney about the client's physical or mental condition
requires the assistance of a physician to interpret the client's condition to the attorney, the
knowledge obtained by the physician is protected under the attorney-client privilege.
When you are an attorney working on a case and using outside investigators, the question
is whether or not those communications relating to the third parties are privileged. The
answer is yes if the conversation is part of the case.
F. Crime- Fraud Exception (Clark v. State)
1. D admitted to murdering his wife. The lawyer then told D to "get rid of the weapon."
2. Public policy does not require protection of a person who takes counsel on how he can
safely commit a crime. This also applies to one who seeks advice on how to destroy
evidence. This is not helping D to make or prepare his legal defense. Such advice is
beyond the legitimate course of professional employment and is not privileged.
If the communication is heard by someone else, the privilege does not apply. Is this the
rule now? It has changed. You need to demonstrate that you intended the conversation to
be confidential and the failure to take precautions reflects a conclusion that you did not
intend to keep the conversation confidential. FRE 503 talks about this.
The other issue in this case is the instruction of getting rid of the weapon. When the
attorney tells this to the client, what is triggered is the crime-fraud exception. Where
conversations between an attorney and client are talking about committing a crime in the
future, this conversation is not privileged. A conversation may talk about conduct that
already occurred and what can be done about it in regards to offense, but it can also talk
about what to do in the future arising out of a past event. If that means committing a
crime in the future, this is an exception to the rule. What is the problematic of the case is
how bad the attorney is.

G. In camera review is permitted, if there is a factual basis to have a good faith belief, to
determine is evidence falls under Crime-Fraud exception (US v. Zolin)
1. Tape recordings between client and lawyer. P sought in camera review to determine if
admissible.
2. In camera review is not intended to give opponents of the privilege groundless fishing
expeditions. Because in camera review is less intrusive than public disclosure, the
evidentiary threshold for triggering in camera review is less stringent than that needed to
overcome the privilege.
3. In camera review does require a showing of a factual basis adequate to support a good
faith belief by a reasonable person that in camera review of the materials may reveal
evidence to establish the claim that the crime-fraud exception applies. Once that showing
is made, the district court has discretion to grant or deny in camera review.
What information can be looked at? This applies to rule 104. The person can use non-
privileged evidence to show their support for in-camera review.

Character Evidence
I. Rule 404. Character Evidence Not Admissible To Prove Conduct; Exceptions; Other Crimes
a) Character evidence generally
Evidence of a person's character or a trait of character is not admissible for the purpose of
proving action in conformity therewith on a particular occasion, except:
(1) Character of accused - In a criminal case, evidence of a pertinent trait of character
offered by an accused, or by the prosecution to rebut the same, or if evidence of a trait
of character of the alleged victim of the crime is offered by an accused and admitted
under Rule 404 (a)(2), evidence of the same trait of character of the accused offered by
the prosecution;
(2) Character of alleged victim - In a criminal case, and subject to the limitations imposed
by Rule 412, evidence of a pertinent trait of character of the alleged victim of the crime
offered by an accused, or by the prosecution to rebut the same, or evidence of a character
trait of peacefulness of the alleged victim offered by the prosecution in a homicide case to
rebut evidence that the alleged victim was the first aggressor;
(3) Character of witness - Evidence of the character of a witness, as provided in rules
607, 608, and 609.

(b) Other crimes, wrongs, or acts


Evidence of other crimes, wrongs, or acts is not admissible to prove the character of a
person in order to show action in conformity therewith. It may, however, be admissible
for other purposes, such as proof of motive, opportunity, intent, preparation, plan,
knowledge, identity, or absence of mistake or accident, provided that upon request by the
accused, the prosecution in a criminal case shall provide reasonable notice in advance of
trial, or during trial if the court excuses pretrial notice on good cause shown, of the
general nature of any such evidence it intends to introduce at trial.
o KIPPOMIA
Knowledge
Intent
Plan
Preparation
Opportunity
Motive
Identity
Absence of mistake or accident.

II. Rule 405. Methods of Proving Character


(a)Reputation or opinion.
o In all cases in which evidence of character or a trait of character of a person is
admissible, proof may be made by testimony as to reputation or by testimony in
the form of an opinion. On cross-examination, inquiry is allowable into relevant
specific instances of conduct.
(b) Specific instances of conduct.
o In cases in which character or a trait of character of a person is an essential
element of a charge, claim, or defense, proof may also be made of specific
instances of that person's conduct.
III. Rule 404 in Depth
A. Evidence of character trait is generally not admissible to show action in conformity with the
trait on a particular occasion.
1. Rationale- Whatever relevance such evidence might have is outweigh by the risks of
undue prejudice and confusion of the issue.
2. Methods of proving character
o a. Personal opinion. One or more persons may offer their personal opinion (based
on observation) as to some person's character for honesty or some other trait.
o b. Reputation. Testimony as to a person's "general reputation" in the community
may be given, whether the witness actually knows the person in question or not.
o C. Specific acts. Evidence may be given of specific acts that a person has done
which would lead to inferences as to his character.

B. 404 (a) Exceptions to General Rule


1. As a matter of fairness, the accused in a criminal prosecution may always introduce
evidence of his good character to show the improbability that he committed the crime
charged. This is true whether or not accused takes the stand.
o a. Reputation and Opinion evidence are admissible, specific acts are not.
o b. Michelson v. US- D was being tried for bribing a public official. D claimed
that he had been entrapped and called witnesses to testify to his good reputation.
On cross-examination the prosecutor asked each of D's character witnesses if he
had heard that D had been arrested 20 years earlier for buying stolen goods.
i. The prosecution cannot introduce character evidence UNLESS the
defendant opens the door.
ii. When a defendant puts his reputation in issue, and calls character
witnesses, the prosecution may question them to determine the extent of
their knowledge of the defendant. This includes questions concerning
specific bad acts, even if they are not related to the crime for which the
defendant is on trial
What kind of character trait was he claiming? He was trying to prove
truthfulness and being a law-abiding citizen. The nature of the testimony
of the five witnesses was reputation testimony. When you examine the
testimony, the foundation laid for events in question was that these
witnesses knew him for 30 years and talked to other people that knew
Michelson for that long. Setting foundation of the relationship is important
before talking about the relationship.
Can a character witness be asked about the arrest of the defendant, given
testimony of a pertinent character trait? It is permissible to ask about
hearing the arrest when they meet the good-faith belief ground. They can
do this because it goes to the reliability of the witness. If they testify about
someones reputation, this question tests the basis of knowledge of the
character witness. Testifying you live in the community for 30 years and
known Michelson for 30 years and that his reputation is good, prosecution
can ask if that person heard about the arrest. If you say you havent heard
about it, in the contexting of testing the knowledge shows you do not
know. If you did know, it brings to question your opinion of Michelson as
truthful. It cant be used against Michelson, but can be used to test the
credibility of the character witness.
Pg,101 A character witness may be cross-examined as to an arrest whether
or not it culminated in a conviction, according to the overwhelming weight
of the authority.

C. 404 (b) Evidence falling outside the general rule Applies to Criminal AND CIVIL
1. Other crimes and acts are admissible when offered to show something other than
character. (See KIPPOMIA) Character can then be used as circumstantial evidence from
which an inference can be drawn as to the existence or nonexistence of some fact which
is in issue.
2. Knowledge- Cleghorn v. New York Central & Hudson River Railway
o a. Train accident caused by negligent switchman. Negligence suit. P sought to
introduce evidence of switchmans drinking habits to show that D knew or should
have known about the switchman's drinking problem.
o b. Whether or not D knew about the switchman's propensity to drink was at issue
because P is seeking exemplary damages. Therefore, the evidence was properly
admitted.
3. Identity- US v. Carrillo
o a. D charged with selling heroin. Said it wasnt him, was two blocks down the
street. Before trial, D presented a motion in limine to exclude evidence of two
other sales of heroin by D. Trial court said if D raised the issue of identity, the
government could call officers to testify about the prior offenses
o b. Overruled: The identity exception of Federal Rule 404(b) has a limited scope. It
does not allow admission of extrinsic acts that are merely similar, but only those
that have such a high degree of similarity as to mark the specific offense as the
handiwork of the accused; i.e., the modus operandi method of proving identity. In
this case, not similar enough.
o Extrinsic evidence is evidence brought in outside this case. The theory of
admissibility is that it will prove identity since identity is an issue. But the case
says that if you are using identity as a basis to enter 404(b) evidence, what is the
quantum of evidence needed? Handiwork is what they turn to. This can include
signature of the defendant, but more broadly a persons way of doing things or
modus operandi. The way the defendant carried out the crime here, based on the
way he handled previous events, proves his modus operandi in the commission of
the crime.
o The evidence is not admissible because in the end the fact that he used a balloon
to sell drugs is common in the drug trade and used by other people. The area that
he was selling drugs in was known to have a high volume of drug trafficking so it
wasnt unusual to occur in this area.
4. Motive- US v. Cunningham
o a. D nurse charged with replacing Demerol with a saline solution in several
syringes. At her trial, the court allowed evidence that D's nurse's license had been
suspended for taking Demerol from the hospital she then worked at, as well as
evidence that she falsified the results of her drug tests and evidence of her
addiction to Demerol. However, the court excluded evidence of her previous
conviction for stealing Demerol.
o b. D's prior conviction could not be used to show that D was likely to have stolen
Demerol in this case. But the other evidence about her prior addiction could be
used to show her motive for committing the charged crime.
o c. In some situations, a motive to commit a crime may be revealed by past
commission of the same crime. When this motive evidence overlaps with
propensity, the judge must be careful to protect against a jury's improper use of
the evidence. (Prejudice)
i. License suspension provided background and supported an inference
that she falsified the test results to keep working at the hospital where she
had easy access to Demerol, which in tum established a motive to tamper
with the Demerol syringes in this case.
The issue was whether the district judge abused his discretion. It was
admitted to prove her motive of stealing Demerol, not to prove that she
actually stole them. Addiction goes towards propensity. What is important
is the instruction to the jury. The evidence of the conviction was not
admitted because it was excessive and the addiction and suspension as
enough.
The other evidence of misconduct was admissible because it went towards
motive. Past history with dealing with Demerol and has an addiction may
provide insight to motive as to why she is the one who committed the
crime in question.
5. Preliminary finding as to criminal nature of prior act not required - Huddleston v. US
o a. D allegedly possessed and sold stolen video cassette tapes knowing that they
were stolen. The district court admitted similar act evidence consisting of
testimony of a store owner and an FBI agent who testified that D had offered to
sell them large quantities of televisions. Offered to prove knowledge. It is
unknown whether the merchandise was stolen.
o b. 104(a) requires the court to determine preliminary questions concerning the
admissibility of evidence. But this does not require the court to make findings
such as D suggests. Federal Rules 40 I and 402 require admission of relevant
evidence unless the rules provide otherwise. There is no requirement of a
preliminary showing before evidence may be introduced for a proper purpose.
o c. The relevancy requirement of Federal Rule 104(b), which does not require the
court to weigh credibility or make findings, is sufficient control over abuse. It
requires the court to examine all the evidence and decide whether the jury could
reasonably find the conditioned fact -in this case, that the televisions were stolen-
by a preponderance of the evidence.
i. Cicero: The relevancy issue is determined by preponderance standard,
and nothing to limit that issue if conditional relevant. Judge then decides
whether the jury can examine it.
o Why is this a matter of conditional relevancy of 104(b) and not a question for the
judge under 104(a)? The conditional fact is whether these items were stolen,
because if it can be proven by preponderance of evidence by the jury, then the
prior bad acts are admissible. When you examine the relevant sphere of factual
information, you look at current charges, the prior misconduct, and if you can
determine if the items are stolen then you can use it to prove that he knew that the
items in this case were stolen.
o Under Huddleston, the trial court examines all of the evidence current, before, and
after. Most times it will be current and prior evidence. After examining the
evidence, the court decides whether the jury could decide the uncharged
misconduct occurred through the preponderance of the evidence.
Character of the Witness
I. Rule 608. Evidence of Character and Conduct of Witness
(a) Opinion and reputation evidence of character.
The credibility of a witness may be attacked or supported by evidence in the form of
opinion or reputation, but subject to these limitations: (1) the evidence may refer only
to character for truthfulness or untruthfulness, and (2) evidence of truthful character is
admissible only after the character of the witness for truthfulness has been attacked
by opinion or reputation evidence or otherwise.
(b) Specific instances of conduct.
Specific instances of the conduct of a witness, for the purpose of attacking or
supporting the witness' character for truthfulness, other than conviction of crime as
provided in rule 609, may not be proved by extrinsic evidence. They may, however, in
the discretion of the court, if probative of truthfulness or untruthfulness, be inquired
into on cross-examination of the witness (1) concerning the witness' character for
truthfulness or untruthfulness, or (2) concerning the character for truthfulness or
untruthfulness of another witness as to which character the witness being cross-
examined has testified.
The giving of testimony, whether by an accused or by any other witness, does not operate
as a waiver of the accused's or the witness' privilege against self-incrimination when examined
with respect to matters that relate only to character for truthfulness.

II. Rule 609. Impeachment by Evidence of Conviction of Crime


(a) General rule.
For the purpose of attacking the character for truthfulness of a witness,
o (1) evidence that a witness other than an accused has been convicted of a
crime shall be admitted, subject to Rule 403, if the crime was punishable by
death or imprisonment in excess of one year under the law under which the
witness was convicted, and evidence that an accused has been convicted of
such a crime shall be admitted if the court determines that the probative value
of admitting this evidence outweighs its prejudicial effect to the accused; and
o (2) evidence that any witness has been convicted of a crime shall be admitted
regardless of the punishment, if it readily can be determined that establishing
the elements of the crime required proof or admission of an act of dishonesty
or false statement by the witness.
(b) Time limit.
Evidence of a conviction under this rule is not admissible if a period of more than ten
years has elapsed since the date of the conviction or of the release of the witness from
the confinement imposed for that conviction, whichever is the later date, unless the
court determines, in the interests of justice, that the probative value of the conviction
supported by specific facts and circumstances substantially outweighs its prejudicial
effect. However, evidence of a conviction more than 10 years old as calculated herein,
is not admissible unless the proponent gives to the adverse party sufficient advance
written notice of intent to use such evidence to provide the adverse party with a fair
opportunity to contest the use of such evidence.
(c) Effect of pardon, annulment, or certificate of rehabilitation.
Evidence of a conviction is not admissible under this rule if (1) the conviction has
been the subject of a pardon, annulment, certificate of rehabilitation, or other
equivalent procedure based on a finding of the rehabilitation of the person convicted,
and that person has not been convicted of a subsequent crime that was punishable by
death or imprisonment in excess of one year, or (2) the conviction has been the
subject of a pardon, annulment, or other equivalent procedure based on a finding of
innocence.
(d) Juvenile adjudications.
Evidence of juvenile adjudications is generally not admissible under this rule. The
court may, however, in a criminal case allow evidence of a juvenile adjudication of a
witness other than the accused if conviction of the offense would be admissible to
attack the credibility of an adult and the court is satisfied that admission in evidence
is necessary for a fair determination of the issue of guilt or innocence.
(e) Pendency of appeal.
The pendency of an appeal therefrom does not render evidence of a conviction
inadmissible. Evidence of the pendency of an appeal is admissible.

III. 608 in depth


A. Whenever a witness (party or nonparty) takes the stand, he puts his character for
honesty and veracity in issue. He can therefore be impeached by evidence that his character is
such that he may lie under oath. The difficult question is what evidence is admissible for this
purpose.
1. Prior bad acts. Misconduct that has not been the subject of a criminal conviction
may still reflect on a witness's veracity.
o a. General view. Most courts allow cross-examination as to prior bad acts if
they are clearly probative of veracity and do not involve unreasonable risks of
prejudice, confusion, etc. The judge has discretion in this area. 608(b)
However, the examiner must be satisfied by the witness's answers; she cannot
introduce extrinsic evidence to prove the past misconduct.
2.Prior Acts of Intentional Falsehood - US v. Owens
o a. Husband shot wife, said was cleaning gun. During cross-examination, D
was asked whether he had omitted from his application for appointment as a
Warrant Officer the facts that he had been convicted of possessing marijuana
and paraphernalia, had been arrested for assault and battery on his second
wife, and had been convicted for carrying a pistol without a permit.
o b. Evidence of D's prior convictions and arrests, and the underlying conduct,
could not be admitted to show D's prior criminal disposition or untruthful
character, but it could be admitted to impeach D by demonstrating a prior act
of intentional falsehood under oath under 608(b). BUT have to weigh
probative value against prejudice.
o As for the first evidence, it was admitted to the fact that he omitted it from his
application shows that he was more likely to lie. Standing alone, if the
prosecution offered this prior conviction into evidence (marijuana possession),
would that be admissible? No because this kind of conviction has little to do
with the ability to tell the truth, which would make it inadmissible. This
simply has small bearing on the ability to tell the truth.
o As to the arrests on the assault and battery on the wife, does that conduct
reflect the ability to tell the truth? No it does not.
o For the possession of a weapon without a permit is closer because presumably
if Owens were approached he would have lied. You need more underlying
circumstances. Without more facts, it would probably be excluded.
o The first and third incidents should be set forth fully as to get a clear
understanding of the nature of the deception. As for the assault and battery on
the second wife, the trial court judges should not have admitted it, but it was a
harmless error so the judgment still stands. The judge was right saying it is
unduly prejudicial because the assault is related to his current murder of his
wife, and also the lying can be proved by the first and third crimes.
3. Extrinsic Evidence on Cross- US v. Drake
o a. D charged with fraud. Prosecutor impeached his testimony with prior
inconsistent statements. The prosecutor also asked questions that referred to
the contents of school records not in evidence.
o b. Although the questions did refer to records not in evidence, cross-
examination questions alone cannot constitute extrinsic evidence.
i. The questions were arguably improper because they assumed facts
not in evidence. In this case, however, D had already been impeached
on a closely related issue through inconsistent statements.
IV. 609 in depth
A. The early common law rule was that a person who had been convicted of a felony, or
any misdemeanor involving dishonesty, was incompetent to testify at all. This rule has been
abandoned, but proof of conviction of certain crimes may be used to impeach a witness.

B. To determine admissibility Cicero says:


1. First look at 609 (a) (2)- laxer standard. If crime involves dishonesty or false
statements, automatically comes in. If not, then look to:
2. 609 (a) (1)
o a. Apply the special balancing test:
i. Special Balancing Test- if the probative value of the evidence
SUBSTANTIALLY outweighs prejudice.
- Stricter than a traditional 403 Test, has to SUBSTANTIALLY
outweigh.
- Policy: encourages the accused to testify.
o b. THEN apply Slowman factors to determine prejudice under Special
balancing test:
i. The impeachment value of the prior crime.
ii. The point in time of the conviction and the witness subsequent
history.
iii. The similarity b/w the past crime and the charged crime.
iv. The importance of the Ds testimony.
v. The centrality of the credibility issue.

C. Which crimes. The courts differ as to which crimes constitute grounds for
impeachment.
1. 609 allows impeachment by any crime involving dishonesty or false statement
(veracity), regardless of whether a felony or misdemeanor, and by any felony (crime
punishable by death or imprisonment in excess of one year), as long as the court
determines that the probative value of admitting this evidence outweighs its
prejudicial effect to the defendant.
2. Prior Conviction no involving dishonesty- US v Sanders
o a. D convicted of assault, filed motion in limine to exclude evidence of prior
convictions. (Prior assault and contraband possession)
o b. Admission of prior similar offenses may do little to impeach the testifying
defendant's credibility, but it can obviously prejudice him. The general rule is
that evidence of similar offenses for impeachment purposes under Federal
Rule 609 should be admitted sparingly, if at all. In this case, the court should
not have admitted the prior convictions under Federal Rule 609.
i. Rule 404(b) does not allow evidence of other crimes to prove the
character of the defendant to show action in conformity therewith. In
this case, D's prior convictions prove only criminal disposition.
o The evidence is not admissible under 609(a)(1) because it is too prejudicial
because it was too similar to the same charge. Speak about limiting the use of
the evidence will not make a difference because the potential of jury misuse is
too high based on 609(a)(1).
3. Federal Courts do not have to apply balancing test in certain situations US v. Wong
o a. D was charged with mail fraud and racketeering. He had two prior mail
fraud convictions. Before testifying, D moved for preclusion of the
convictions for impeachment purposes
o b. 609(a) distinguishes between convictions for crimes punish- able by
imprisonment of more than one year and convictions for crimen falsi, or
crimes involving dishonesty or false statement. The former convictions are
admissible only after the court applies a balancing test; admission of the latter
is ostensibly mandatory.
o C. 403 was not intended to override more specific rules. The legislative
history of Federal Rule 609(a) indicates an intent that prior convictions for
crimen falsi may always be used to impeach. Therefore, the balancing test of
Federal Rule 403 does not apply to impeachment by crimen falsi convictions.
The issue on appeal was if the lower court have discretion disclose
evidence of prior convictions of dishonesty based on undue evidence?
No. When impeaching the witness the questioning party can ask about
prior convictions involving dishonesty and false statements, but
District Court cannot exclude it based on a balancing test.
Prosecution may put a crime under 609(a)(2) because judge cannot
exclude such evidence if it fits there. When the conviction involves
dishonesty and false statements, then it will be admitted by the District
Court.
4. Bank robbery does not involve "dishonesty" US v Brackeen
o a. Court allowed the prosecutor to impeach D with his guilty pleas under
Federal Rule 609(a)(2), on the rationale that bank robbery involves dishonesty
or false statement.
o b. D's unarmed bank robberies did not involve any false statements and were
not committed by fraudulent or deceitful means. Thus, they would come under
Federal Rule 609(a)(2) only if bank robbery is per se a crime of "dishonesty."
o c. The term "dishonesty" has many meanings, and can include stealing. But its
narrower meaning requires deceitful behavior-a disposition to lie, cheat, or
defraud.
o Brackeen was on trial for committing robbery. The evidentiary issue in this
case was whether a robbery is a crime of dishonesty. There was 3 bank
robberies on 3 different days.
o Are robberies dishonest crimes? The trial judge is admitting the evidence
because of crimen falsi. Judge doesnt want to admit it under 609 because it
did not involve fraudulent or deceitful means. 609(a)(1) was not used because
you need to pass a balancing test. So he let it in 609(a)(2) cause theft sounds
like a crime of dishonesty. The court lists some crimen falsi crimes pg. 494
and they describe crimes with false statements is a narrow definition of
dishonesty, deceit, and false statement crimes.
5. Defendant must testify to preserve claim of error on admission of prior conviction.
Luce v. US
o a. D was tried for conspiracy and possession of cocaine with intent to
distribute. D moved for a ruling to prevent the prosecution from using a prior
conviction to impeach him if he testified, but he did not commit to testify if
the court issued the ruling, and he did not describe what his testimony would
be.
o b. The appellate court must have a complete record in order to weigh the
probative value of a prior conviction against the prejudicial effect to D. When
D does not testify, the possible harm of the ruling is speculative.
o c. Reviewing courts can only re- view a 609(a) claim when D preserves the
claim by testifying.
o Petitioner was indicted of possession of cocaine with intent to distribute. He
moved to preclude the government from using a 1974 conviction if he were to
testify. Possession of a controlled substance was the prior charge. The district
court found it admissible under 609(a). The court in ruling on the motion did
say that the trial testimony could affect the evidentiary ruling, so if the
defendant limited his testimony to just fleeing from officers, then the prior
conviction would not be admitted. But if he denied prior conviction with
crime, they could use the prior conviction to impeach.
o It is clear, of course, that had the petitioner testified and been impeached by
evidence of a prior conviction, the District Courts decision to admit the
impeachment evidence would have been reviewable on appeal along with any
other claims of error.
o When the defendant does not testify, the reviewing court also has no way of
knowing whether the Government would have sought to impeach with the
prior conviction.

Habit
I. Rule 406. Habit; Routine Practice
Evidence of the habit of a person or of the routine practice of an organization,
whether corroborated or not and regardless of the presence of eyewitnesses, is
relevant to prove that the conduct of the person or organization on a particular
occasion was in conformity with the habit or routine practice. habit is important bc if
the door is shut on charcter evidence then you can bring it in as a habit
Threshold: How often it happens, the more it occured the more admissible.

II. 406 in Depth


A. Character v. Habit- Character refers to a persons general propensity. Character traits
influence behavior. On the other hand, habit evidence refers to a persons routine reactions in
particular situations. Can be shown that D is the aggressor bc D always intiates fights...

B. Requirements- the habit must be specific, routine (preformed without deliberation) and
continuous.
1. Specific behavior evidence used most often, opinion evidence can also be used.

C. Perrin v. Anderson-
1. Assault case. Four police officers were permitted, over P's objection, to testify that
they had been involved in violent encounters with D. Show D had a violent response
whenever contacted by Police officers. Use to show D was first aggressor in the fight.
The court ruled that the lower court was right in allowing in the evidence.
2. Under 404(a), character evidence used to show that an individual acted in
conformity with his character is subject to strict limitations. The rule on its face
applies only to criminal cases, but it may also apply to a civil case that, as here, is in
nature criminal.
3. Specific instances of conduct may not be used unless character is in issue in the
strict sense, when it is a material fact such as it would be in a defamation case. This
exception does not apply here.
4. . A habit is a regular practice of meeting a particular kind of situation with a certain
type of conduct or a reflex behavior in a specific set of circumstances. There were
sufficient incidents of Perrin's violent reactions to permit introduction of the specific
acts to show habit.
o This is a civil action because it is similar to issues raised in criminal context,
so character evidence should apply. It would be improper under 404(a)
because there is a ban on character evidence. Perrin says this is proving his
character. But defendant can choose to place into record evidence of a
pertinent character trait of the person. Reputation or opinion would be the
correct way to do this because you cannot use specific instances of conduct.
You can use specific instances of conduct under 405(b) where character or
trait of a person is essential to a charge.
o If the cause of action requires proof of good moral character that means
character is an issue. In those circumstances where character is at issue, it can
be proven by all 3 methods of proof: reputation, opinion, and specific
instances of conduct. But the court says this isnt that type of case where
character is in issue; it is character being used circumstantially.
o The alleged habit in the case is the victim being violent when he sees an
officer. This doesnt sound like a habit. The court says the nature of the habit
is extraordinary. Are 3 instances sufficient? Or 5? We have to consider this.
No evidence to rebut the violent run-ins with the police.
o 406- Evidence inadmissible based on character evidence can be admitted
through habit. Admissible to show action in conformity- not used or for
corroboration.

D. Halloran v. VA chemicals
1. Negligence case, can exploded while mechanic installing. D tried to show it was
Ps habit to not follow standard protocol while installing. It was remitted to the lower
court on the issue of liability.
2. When negligence is the issue, courts resist permitting evidence of specific acts of
carelessness or carefulness to create an inference that such conduct was repeated
when like circumstances were again presented.
3. However, proof of a deliberate repetitive practice by one in complete control of the
circumstances is admissible because it is so highly probative of an unexplained
occurrence.
o You cant offer evidence of prior instances of careless to prove an inference
that the person in question acted negligently in this case. This case involves an
issue where the defendant claims the victim is contributorily negligent
because the way he would routinely deal with the coolant. That evidence will
be admissible if there is sufficient sampling to the response to that type of
situation.

S-ar putea să vă placă și